SlideShare a Scribd company logo
1 of 102
<a href="https://www.freepik.com/free-photo/eco-message-bottle-
concept_31760605.htm#fromView=search&page=1&position=3&uuid=faa0320f-8519-42e1-a493-88cbb342ec46">Image by freepik</a>
ZOOLOGY QUESTION BANK
FOR THE NEET-UG 2024 EXAM
Dr. Shivankan Kakkar, MBBS, MD (Gold Medal)
FLASHCARDS
This is a set of 50 flashcards that
contains zoology questions from the
NEET-UG 2023 exam.
LINKED LEARNING
You can use these flashcards to test your
knowledge and prepare for the exam.
Dr. Shivankan Kakkar, MBBS, MD (Gold Medal)
SECTION A
1. Which of the following statements are correct regarding female reproductive
cycle?
A. In non-primate mammals cyclical changes during reproduction are called oestrus
cycle.
B. First menstrual cycle begins at puberty and is called menopause.
C. Lack of menstruation may be indicative of pregnancy.
D. Cyclic menstruation extends between menarche and menopause.
Choose the most appropriate answer from the options given below.
(1) A and B only
(2) A, B and Conly
(3) A, C and D only
(4) A and D only
Answer (3)
Dr. Shivankan Kakkar, MBBS, MD (Gold Medal)
• Statement A: This statement is true. In non-primate mammals, cyclical changes
during reproduction are called oestrus cycle. This is a type of reproductive cycle
in which females are sexually receptive only during a specific period called
oestrus or heat1
. During oestrus, the female exhibits behavioural and
physiological changes that attract males for mating1
.
• Statement B: This statement is false. The first menstrual cycle begins at
puberty and is called menarche, not menopause. Menarche is the onset of
menstruation in females, which usually occurs between the ages of 11 and 152
.
Menopause is the cessation of menstruation in females, which usually occurs
between the ages of 45 and 552
.
• Statement C: This statement is true. Lack of menstruation may be indicative of
pregnancy, as one of the earliest signs of pregnancy is a missed period3
.
However, there may be other causes of amenorrhea (absence of menstruation),
such as hormonal imbalance, stress, illness, or medication3
.
• Statement D: This statement is true. Cyclic menstruation extends between
menarche and menopause, which are the two milestones that mark the
beginning and the end of the reproductive years in females2
. The menstrual
cycle is a monthly cycle of hormonal and physiological changes that prepare the
female body for pregnancy2
.
Therefore, the correct answer is (3) A, C and D only.
Dr. Shivankan Kakkar, MBBS, MD (Gold Medal)
2. Given below are two statements: one is labelled as Assertion A and other is
labelled as Reason R.
Assertion A : Amniocentesis for sex determination is one of the strategies of
Reproductive and Child Health Care Programme.
Reason R : Ban on amniocentesis checks increasing menace of female foeticide.
In the light of the above statements, choose the correct answer from the options
given below.
(1) Both A and R are true and R is NOT the correct explanation of A.
(2) A is true but R is false.
(3) A is false but R is true.
(4) Both A and R are true and R is the correct explanation of A.
Answer (3)
Dr. Shivankan Kakkar, MBBS, MD (Gold Medal)
• Assertion A: This statement is false. Amniocentesis is not a strategy of
Reproductive and Child Health Care Programme, but a prenatal diagnostic
technique that can detect the sex and genetic abnormalities of the fetus1
.
However, this technique has been misused for sex determination and female
foeticide, which is the deliberate abortion of a female fetus23
. Therefore,
amniocentesis is not a part of the reproductive and child health care programme,
but a tool that can be abused for gender discrimination.
• Reason R: This statement is true. Ban on amniocentesis checks the increasing
menace of female foeticide, as it prevents the illegal use of this technique for
sex selection and abortion. The Indian government has passed the Pre-
Conception and Pre-Natal Diagnostic Techniques (PCPNDT) Act in 1994 to
prohibit and punish the misuse of amniocentesis and other prenatal diagnostic
methods for sex determination and female foeticide4
. The ban on amniocentesis
is aimed at protecting the rights and dignity of the girl child and ensuring
gender equality.
Therefore, the correct answer is (3) A is false but R is true.
Dr. Shivankan Kakkar, MBBS, MD (Gold Medal)
3. Match List I with List II. List I List II (Interacting species) (Name of interaction)
A. A Leopard and a Lion in a forest/grassland I. Competition
B. A Cuckoo laying egg in a Crow’s nest II. Brood parasitism
C. Fungi and root of a higher plant in Mycorrhizae III. Mutualism
D. A cattle egret and a Cattle in a field IV. Commensalism
Choose the correct answer from the options given below.
(1) A-I, B-II, C-IV, D-III
(2) A-III, B-IV, C-I, D-II
(3) A-II, B-III, C-I, D-IV
(4) A-I, B-II, C-III, D-IV
Answer (4)
Dr. Shivankan Kakkar, MBBS, MD (Gold Medal)
The matching of the interacting species with the name of interaction is as follows:
• A. A Leopard and a Lion in a forest/grassland: This is an example of
competition (I), which is a type of interaction where two or more species use
the same limited resource, such as food, water, space, or mates, and negatively
affect each other’s fitness12
. Leopards and lions are both carnivorous predators
that hunt similar prey and may compete for food in the same habitat3
.
• B. A Cuckoo laying egg in a Crow’s nest: This is an example of brood
parasitism (II), which is a type of interaction where one species lays its eggs in
the nest of another species and relies on the host to raise its offspring, often at
the expense of the host’s own offspring12
. Cuckoos are notorious brood
parasites that exploit various species of birds, such as crows, by laying eggs that
mimic the host’s eggs and removing or destroying some of the host’s eggs4
.
• C. Fungi and root of a higher plant in Mycorrhizae: This is an example of
mutualism (III), which is a type of interaction where two or more species
benefit from each other and enhance each other’s fitness12
. Mycorrhizae are
symbiotic associations between fungi and the roots of higher plants, where the
fungi help the plants absorb water and minerals from the soil, and the plants
provide the fungi with organic nutrients.
• D. A cattle egret and a Cattle in a field: This is an example of commensalism
(IV), which is a type of interaction where one species benefits from another
species, but the other species is neither helped nor harmed12
. Cattle egrets are
birds that follow cattle and other large herbivores in fields, and feed on the
Dr. Shivankan Kakkar, MBBS, MD (Gold Medal)
insects that are disturbed by the cattle’s grazing. The cattle egrets gain access to
food, but the cattle are unaffected by their presence.
Therefore, the correct answer is (4) A-I, B-II, C-III, D-IV.
Dr. Shivankan Kakkar, MBBS, MD (Gold Medal)
4. Vital capacity of lung is _________.
(1) IRV + ERV + TV + RV
(2) IRV + ERV + TV –RV
(3) IRV + ERV + TV
(4) IRV + ERV
Answer (3)
Dr. Shivankan Kakkar, MBBS, MD (Gold Medal)
Vital capacity of lung is the total amount of air that can be exhaled after a maximum
inhalation. It is equal to the sum of inspiratory reserve volume (IRV), tidal volume
(TV), and expiratory reserve volume (ERV). It is an important measure of a
person’s respiratory health. A decreased vital capacity is an indication of restrictive
lung disease, where the lungs cannot expand fully.
One of the links that explains this is here. This is a web page that talks about the
lung volumes and capacities, such as tidal volume, residual volume, vital capacity,
inspiratory capacity, etc.
Dr. Shivankan Kakkar, MBBS, MD (Gold Medal)
5. Which one of the following common sexually transmitted diseases is completely
curable when detected early and treated properly?
(1) Gonorrhoea
(2) Hepatitis-B
(3) HIV Infection
(4) Genital herpes
Answer (1)
Dr. Shivankan Kakkar, MBBS, MD (Gold Medal)
Gonorrhoea is a bacterial infection that can be cured with antibiotics. Hepatitis-B,
HIV infection, and genital herpes are viral infections that are not completely
curable.
One of the links that explains this is here.
Dr. Shivankan Kakkar, MBBS, MD (Gold Medal)
6. Match List I with List II.
List I List II
A. CCK I. Kidney
B. GIP II. Heart
C. ANF III. Gastric gland
D. ADH IV. Pancreas
Choose the correct answer from the options given below :
(1) A-III, B-II, C-IV, D-I
(2) A-II, B-IV, C-I, D-III
(3) A-IV, B-II, C-III, D-I
(4) A-IV, B-III, C-II, D-I
Answer (4)
Dr. Shivankan Kakkar, MBBS, MD (Gold Medal)
The correct answer is (4) A-IV, B-III, C-II, D-I. Here is a brief explanation of the
matching of the hormones with their sources:
• CCK (A): CCK stands for cholecystokinin, a hormone that is secreted by the
cells of the duodenum and jejunum in response to the presence of fats and
proteins in the small intestine. It stimulates the gallbladder to release bile and
the pancreas to secrete digestive enzymes. It matches with Pancreas (IV)12
.
• GIP (B): GIP stands for gastric inhibitory peptide, a hormone that is secreted by
the cells of the duodenum in response to the presence of glucose and fats in the
small intestine. It inhibits the secretion of gastric acid and motility, and
stimulates the secretion of insulin. It matches with Gastric gland (III)13
.
• ANF ©: ANF stands for atrial natriuretic factor, a hormone that is secreted by
the cells of the atria of the heart in response to high blood pressure or volume. It
causes the kidneys to excrete more sodium and water, and lowers the blood
pressure. It matches with Heart (II)4
.
• ADH (D): ADH stands for antidiuretic hormone, a hormone that is secreted by
the posterior pituitary gland in response to low blood volume or high blood
osmolarity. It causes the kidneys to reabsorb more water, and increases the
blood volume and pressure. It matches with Kidney (I)4
.
Dr. Shivankan Kakkar, MBBS, MD (Gold Medal)
7. Match List I with List II.
List I List II
A. Ringworm I. Haemophilus influenzae
B. Filariasis II. Trichophyton
C. Malaria III. Wuchereria bancrofti
D. Pneumonia IV. Plasmodium vivax
Choose the correct answer from the options given below :
(1) A-II, B-III, C-I, D-IV
(2) A-III, B-II, C-I, D-IV
(3) A-III, B-II, C-IV, D-I
(4) A-II, B-III, C-IV, D-I
Answer (4)
Dr. Shivankan Kakkar, MBBS, MD (Gold Medal)
The correct answer is (4) A-II, B-III, C-IV, D-I. Here is a brief explanation of the
matching of the diseases with their causes:
• Ringworm (A): Ringworm is a common skin infection that is caused by a
fungus. It’s usually an itchy, circular rash with clearer skin in the middle. It
matches with Trichophyton (II), which is one of the three types of fungi that
cause ringworm12345
.
• Filariasis (B): Filariasis is a parasitic disease that is caused by infection with
thread-like worms called filariae. It can cause swelling of the limbs, genitals,
and breasts, and can lead to elephantiasis. It matches with Wuchereria
bancrofti (III), which is the most common cause of lymphatic filariasis.
• Malaria ©: Malaria is a life-threatening disease that is caused by infection with
a parasite called Plasmodium. It is transmitted by the bite of an infected
mosquito. It can cause fever, chills, headache, nausea, vomiting, and other
symptoms. It matches with Plasmodium vivax (IV), which is one of the five
species of Plasmodium that cause malaria in humans.
• Pneumonia (D): Pneumonia is an infection that inflames the air sacs in one or
both lungs. It can be caused by bacteria, viruses, fungi, or other organisms. It
can cause cough, fever, difficulty breathing, and chest pain. It matches with
Haemophilus influenzae (I), which is one of the common bacterial causes of
pneumonia.
Dr. Shivankan Kakkar, MBBS, MD (Gold Medal)
8. Match List I with List II.
List I List II
A. P-wave I. Beginning of systole
B. Q-wave II. Repolarisation of ventricles
C. QRS complex III. Depolarisation of atria
D. T-wave IV. Depolarisation of ventricles
Choose the correct answer from the options given below :
(1) A-IV, B-III, C-II, D-I
(2) A-II, B-IV, C-I, D-III
(3) A-I, B-II, C-III, D-IV
(4) A-III, B-I, C-IV, D-II
Answer (4)
Dr. Shivankan Kakkar, MBBS, MD (Gold Medal)
The correct answer is (4) A-III, B-I, C-IV, D-II. Here is a brief explanation of the
matching of the ECG waves with their corresponding events:
• P-wave (A): The P-wave reflects atrial depolarization, which is the electrical
activation of the atria. It occurs before the atria contract and push blood into the
ventricles. It matches with Depolarisation of atria (III)1
.
• Q-wave (B): The Q-wave is the first negative deflection of the QRS complex. It
represents the depolarization of the interventricular septum, which is the
muscular wall that separates the left and right ventricles. It matches with
Beginning of systole (I)2
, which is the phase of the cardiac cycle when the
ventricles contract and eject blood.
• QRS complex ©: The QRS complex is the most prominent feature of the ECG.
It consists of three waves: Q, R, and S. It represents the depolarization of the
ventricles, which is the electrical activation of the ventricular myocardium. It
matches with Depolarisation of ventricles (IV)2
.
• T-wave (D): The T-wave is the positive deflection that follows the QRS
complex. It represents the repolarization of the ventricles, which is the
restoration of the electrical potential of the ventricular myocardium after
contraction. It matches with Repolarisation of ventricles (II)2
.
Dr. Shivankan Kakkar, MBBS, MD (Gold Medal)
9. In which blood corpuscles, the HIV undergoes replication and produces progeny
viruses?
(1) B-lymphocytes
(2) Basophils
(3) Eosinophils
(4) T helper cells
Answer (4)
Dr. Shivankan Kakkar, MBBS, MD (Gold Medal)
HIV undergoes replication and produces progeny viruses in T helper cells, which
are a type of white blood cell that helps other immune cells to fight infections.
• HIV - Biology: AQA A Level
Dr. Shivankan Kakkar, MBBS, MD (Gold Medal)
10. Given below are two statements :
Statement I : Low temperature preserves the enzyme in a temporarily inactive state
whereas high temperature destroys enzymatic activity because proteins are
denatured by heat.
Statement II : When the inhibitor closely resembles the substrate in its molecular
structure and inhibits the activity of the enzyme, it is known as competitive
inhibitor.
In the light of the above statements, choose the correct answer from the options
given below :
(1) Both Statement I and Statement II are false.
(2) Statement I is true but Statement II is flase.
(3) Statement I is false but Statement II is true.
(4) Both Statement I and Statement II are true.
Answer (4)
Dr. Shivankan Kakkar, MBBS, MD (Gold Medal)
• Statement I: This statement is true. Low temperature preserves the enzyme in a
temporarily inactive state because it reduces the kinetic energy of the molecules
and slows down the rate of reaction. However, low temperature does not affect
the structure or function of the enzyme, and the enzyme can resume its activity
when the temperature is increased1
. High temperature destroys enzymatic
activity because proteins are denatured by heat. This means that the high
temperature disrupts the bonds that maintain the three-dimensional shape of the
enzyme, and thus alters its active site and prevents it from binding to the
substrate1
.
• Statement II: This statement is true. When the inhibitor closely resembles the
substrate in its molecular structure and inhibits the activity of the enzyme, it is
known as a competitive inhibitor. A competitive inhibitor competes with the
substrate for the same active site of the enzyme, and thus reduces the number of
enzyme-substrate complexes formed2
. The effect of competitive inhibition can
be overcome by increasing the concentration of the substrate, which increases
the chances of the substrate binding to the enzyme2
.
Therefore, the correct answer is (4) Both Statement I and Statement II are true.
Dr. Shivankan Kakkar, MBBS, MD (Gold Medal)
11. Given below are two statements:
Statement I: Ligaments are dense irregular tissue.
Statement II: Cartilage is dense regular tissue.
In the light of the above statements, choose the correct answer from the options
given below:
(1) Both Statement I and Statement II are false
(2) Statement I is true but Statement II is false
(3) Statement I is false but Statement II is true
(4) Both Statement I and Statement II are true
Answer (1)
Dr. Shivankan Kakkar, MBBS, MD (Gold Medal)
The correct answer is (1) Both Statement I and Statement II are false.
Ligaments are not dense irregular tissue, but dense regular tissue. They are
composed of parallel bundles of collagen fibers that connect bones to bones and
provide strength and stability to joints.
Cartilage is not dense regular tissue, but a type of specialized connective tissue. It is
composed of a dense network of collagen and elastic fibers embedded in a firm,
gelatinous ground substance. It is found at the end of joints, the rib cage, the ear, the
nose, in the throat, and between intervertebral disks.
• Dense connective tissue: Cells, fibers and types
• Dense irregular connective tissue: anatomy and function
• Cartilage | Description, Anatomy, & Function
Dr. Shivankan Kakkar, MBBS, MD (Gold Medal)
12. Which of the following are NOT considered as the part of endomembrane
system?
A. Mitochondria
B. Endoplasmic reticulum
C. Chloroplasts
D. Golgi complex
E. Peroxisomes
Choose the most appropriate answer from the options given below:
(1) A,C and E only
(2) A and D only
(3) A,D and E only
(4) B and D only
Answer (1)
Dr. Shivankan Kakkar, MBBS, MD (Gold Medal)
The correct answer is indeed (1) A,C and E only. The endomembrane system is a
group of membranes and organelles in eukaryotic cells that work together to
modify, package, and transport lipids and proteins. It includes the nuclear envelope,
the endoplasmic reticulum, the Golgi complex, lysosomes, endosomes, vesicles, and
the plasma membrane1
. Mitochondria, chloroplasts, and peroxisomes are not part of
the endomembrane system because they have their own membranes and are not
derived from the endoplasmic reticulum2
.
Dr. Shivankan Kakkar, MBBS, MD (Gold Medal)
13. Given below are two statements:
Statement I: A protein is imagined as a line, the left end represented by first amino
acid (C-terminal) and the right end represented by last amino acid (N-terminal).
Statement II: Adult human haemoglobin, consists of 4 subunits (two subunits of a
type and two subunits of ß type.)
In the light of the above statements, choose the correct answer from the options
given below:
(1) Both Statement I and Statement II are false.
(2) Statement I is true but Statement II is false.
(3) Statement I is false but Statement II is true.
(4) Both Statement I and Statement II are true.
Answer (3)
Dr. Shivankan Kakkar, MBBS, MD (Gold Medal)
• Statement I: This statement is false. A protein is imagined as a line, the left end
represented by the first amino acid (N-terminal) and the right end represented
by the last amino acid (C-terminal)12
. The N-terminal is the end of the protein
chain that has a free amino group, while the C-terminal is the end that has a free
carboxyl group12
.
• Statement II: This statement is true. Adult human haemoglobin consists of four
subunits (two subunits of α type and two subunits of β type)34
. Each subunit
contains a heme group, which is a complex of an iron atom and a porphyrin
ring, that can bind oxygen reversibly34
.
Therefore, the correct answer is (3), as statement I is false but statement II is true.
Dr. Shivankan Kakkar, MBBS, MD (Gold Medal)
14. Broad palm with single palm crease is visible in a person suffering from­
(1) Turner’s syndrome
(2) Klinefelter’s syndrome
(3) Thalassemia
(4) Down’s syndrome
Answer (4)
Dr. Shivankan Kakkar, MBBS, MD (Gold Medal)
Some key points on the single transverse palmar crease, also known as the simian
crease:
• A single transverse palmar crease is a single crease that extends across the palm
of the hand, formed by the fusion of the two palmar creases1
.
• It is found at a higher frequency in people with certain disorders, such as Down
syndrome, fetal alcohol syndrome and others1
.
• Down syndrome is a genetic disorder caused by the presence of an extra copy of
chromosome 21. It causes intellectual disabilities, a characteristic facial
appearance, and an increased chance for heart defects and digestive issues2
.
• A broad palm with a single palm crease is a characteristic physical feature
commonly associated with individuals who have Down syndrome3
. However,
not all people with Down syndrome have this feature, and thus it is not a
diagnostic indicator of the disorder1
.
Therefore, the correct answer is (4), as broad palm with single palm crease is visible
in a person suffering from Down syndrome.
Dr. Shivankan Kakkar, MBBS, MD (Gold Medal)
15. Which of the follwoing statements is correct?
(1) Biomagnification refers to increase in concentration of the toxicant at successive
trophic levels.
(2) Presence of large amount of nutrients in water restricts ‘Algal Bloom’
(3) Algal Bloom decreases fish mortality
(4) Eutrophication refers to increase in domestic sewage and waste water in lakes.
Answer (1)
Dr. Shivankan Kakkar, MBBS, MD (Gold Medal)
• Biomagnification is the process of accumulation of certain chemicals in living
organisms to a concentration higher than that occurring in the inorganic, non-
living environment. It can be caused by agriculture, organic contaminants,
industrial activities and mining activities. It has negative impacts on human
health, reproduction, coral reefs and food chain.
• Algal bloom is a rapid increase or accumulation in the population of algae in
freshwater or marine water systems. It is often recognized by the discoloration
in the water from the algae’s pigments. It can be caused by human activities
such as fertilizers, sewage, or industrial wastes, or by natural events such as
floods or erosion. Algal bloom has serious environmental and ecological
consequences, such as reducing biodiversity, depleting oxygen, and creating
dead zones.
• Eutrophication is the process of excessively enriching a water body in
nutrients, primarily phosphorus and nitrogen, leading to the plentiful growth of
simple plant life. This process can significantly disrupt aquatic ecosystems and
is influenced by a variety of factors, including agricultural runoff, industrial
discharge, and urbanization. Eutrophication can result in algal blooms, fish
kills, and loss of aquatic habitats.
Therefore, the correct answer is (1), as biomagnification refers to the increase in
concentration of the toxicant at successive trophic levels. The other statements are
incorrect, as presence of large amount of nutrients in water promotes algal bloom,
algal bloom increases fish mortality, and eutrophication refers to the enrichment of
water body in nutrients, not sewage and waste water.
Dr. Shivankan Kakkar, MBBS, MD (Gold Medal)
16. Match List I with List II.
List I List II
A. Taenia I. Nephridia
B. Paramoecium II. Contractile vacuole
C. Periplaneta III. Flame cells
D. Pheretima IV. Urecose gland
Choose the correct answer from the options given below:
(1) A-I, B-II, C-IV, D-III
(2) A-III, B-II, C-IV, D-I
(3) A-II, B-I, C-IV, D-III
(4) A-I, B-II, C-III, D-IV
Answer (2)
Dr. Shivankan Kakkar, MBBS, MD (Gold Medal)
• Taenia (A): Taenia is a genus of tapeworms. The excretory organs in Taenia
are flame cells (III) which are scattered all over the mesenchyme12345
.
• Paramoecium (B): Paramoecium is a genus of unicellular ciliates. It has a
contractile vacuole (II) that helps regulate the water content within the cell6789
.
• Periplaneta ©: Periplaneta is a genus of cockroaches. It has urecose glands
(IV) as part of its excretory system101112
.
• Pheretima (D): Pheretima is a genus of earthworms. It has nephridia (I) as
part of its excretory system131415161718
.
So, the correct answer is (2) A-III, B-II, C-IV, D-I.
Dr. Shivankan Kakkar, MBBS, MD (Gold Medal)
17. Given below are two statements:
Statement I:Electrostatic precipitator is most widely used in thermal power plant .
Statement II: Electrostatic precipitator in thermal power plant removes ionising
radiations.
In the light of the above statements, choose the most appropriate answer from the
options given below:
(1) Both Statement I and Statement II are incorrect.
(2) Statement I is correct but Statement II is incorrect.
(3) Statement I is incorrect but Statement II is correct.
(4) Both Statement I and Statement II are correct.
Answer (2)
Dr. Shivankan Kakkar, MBBS, MD (Gold Medal)
• Statement I: An electrostatic precipitator (ESP) is a filtration device that is
used to remove fine particles like smoke and fine dust from the flowing gas12345
.
It is the most commonly used device for air pollution control and is widely used
in thermal power plants12345
.
• Statement II: However, electrostatic precipitators do not remove ionizing
radiations67
. They work by using electrostatic charges to attract and trap
particles on charged plates or tubes67
.
So, the correct answer is (2) Statement I is correct but Statement II is incorrect.
Dr. Shivankan Kakkar, MBBS, MD (Gold Medal)
18. Given below are two statements:
Statement I: Vas deferens receives a duct from seminal vesicle and opens into
urethra as the ejaculatory duct.
Statement Il: The cavity of the cervix is called cervical canal which along with
vagina forms birth canal.
In the light of the above statements, choose the correct answer from the options
given below:
(1) Both Statement I and Statement II are false.
(2) Statement I is correct but Statement II is false.
(3) Statement I is incorrect but Statement II is true.
(4) Both Statement I and Statement II are true.
Answer (4)
Dr. Shivankan Kakkar, MBBS, MD (Gold Medal)
• Statement I: The vas deferens receives a duct from the seminal vesicle and
opens into the urethra as the ejaculatory duct12345
. The vas deferens is a
continuation of the cauda epididymis (tail part of epididymis). It is about 40 cm
long and slightly coiled at first but becomes straight as it enters the abdominal
cavity through the inguinal canal. Here, it passes over the urinary bladder,
curves around the ureter and joins a duct from the seminal vesicle and opens
into the urethra as the ejaculatory duct12345
.
• Statement II: The cavity of the cervix is called the cervical canal which along
with the vagina forms the birth canal678910
. The cervical canal communicates
with the uterine cavity via the internal orifice of the uterus (or internal os) and
with the vagina via the external orifice of the uterus (ostium of uterus or
external os). The cervical canal, along with the vagina, forms the birth canal
through which a baby passes during childbirth678910
.
So, the correct answer is (4) Both Statement I and Statement II are true.
Dr. Shivankan Kakkar, MBBS, MD (Gold Medal)
19. Radial symmetry is NOT found in adults of phylum ______.
(1) Hemichordata
(2) Coelenterata
(3) Echinodermata
(4) Ctenophora
Answer (1)
Dr. Shivankan Kakkar, MBBS, MD (Gold Medal)
The correct answer is (1) Hemichordata.
Radial symmetry is NOT found in adults of phylum Hemichordata, a group of
marine deuterostomes that are bilaterally symmetrical.
Dr. Shivankan Kakkar, MBBS, MD (Gold Medal)
20. Match List I with List II .
List I List II
(Cells) (Secretion)
A. Peptic cells I. Mucus
B. Goblet cells II. Bile juice
C. Oxyntic cells III. Proenzyme pepsinogen
D. Hepatic cells IV. HCl & intrinsic factor for absorption of vit B12
Choose the correct answer from the options given below:
(1) A-II, B-I, C-III, D-IV
(2) A-III, B-I, C-IV, D-II
(3) A-II, B-IV, C-I, D-III
(4) A-IV, B-III, C-II, D-I
Answer (2)
Dr. Shivankan Kakkar, MBBS, MD (Gold Medal)
• Peptic cells (A): These cells secrete the proenzyme pepsinogen, which is later
activated to the enzyme pepsin and helps in the digestion of proteins12
. So, they
match with Proenzyme pepsinogen (III)12
.
• Goblet cells (B): These are a specialized type of epithelial cell that secrete
mucins, which are significant components of mucus34
. They are most often
found in the respiratory and gastrointestinal tracts, where they make up part of
the surface epithelium34
. So, they match with Mucus (I)34
.
• Oxyntic cells ©: Also known as parietal cells, these cells secrete hydrochloric
acid and intrinsic factor, which is necessary for the absorption of vitamin B1212
.
So, they match with HCl & intrinsic factor for absorption of vit B12 (IV)12
.
• Hepatic cells (D): These are the main functional cells of the liver and are
involved in protein synthesis, protein storage and transformation of
carbohydrates, synthesis of cholesterol, bile salts and phospholipids, and
detoxification, modification and excretion of exogenous and endogenous
substances. They secrete bile juice1
. So, they match with Bile juice (II)1
.
So, the correct answer is (2) A-III, B-I, C-IV, D-II.
Dr. Shivankan Kakkar, MBBS, MD (Gold Medal)
21. Match List I with List II with respect in human eye.
List I List II
A. Fovea I. Visible coloured portion of eye that regulates diameter of pupil.
B. Iris II. External layer of eye formed of dense connective tissue.
C. Blind spot III. Point of greatest visual acuity or resolution.
D. Sclera IV. Point where optic nerve leaves the eyeball and photoreceptor
cells are absent.
Choose the correct answer from the options given below:
(1) A-IV, B-III, C-II, D-I
(2) A-I, B-IV, C-III, D-II
(3) A-II, B-I, C-III, D-IV
(4) A-III, B-I, C-IV, D-II
Answer (4)
Dr. Shivankan Kakkar, MBBS, MD (Gold Medal)
• Fovea (A): The fovea is a small depression in the retina where visual acuity is
highest. It is the point of greatest visual acuity or resolution (III)12
.
• Iris (B): The iris is the colored part of the eye that controls the amount of light
that enters the eye. It is the visible colored portion of the eye that regulates
the diameter of the pupil (I)12
.
• Blind spot ©: The blind spot is the point in the visual field that corresponds to
the lack of light-detecting photoreceptor cells on the optic disc of the retina
where the optic nerve passes through the optic disc. It is the point where the
optic nerve leaves the eyeball and photoreceptor cells are absent (IV)12
.
• Sclera (D): The sclera is the white outer layer of the eye. It is the external
layer of the eye formed of dense connective tissue (II)12
.
So, the correct answer is (4) A-III, B-I, C-IV, D-II.
Dr. Shivankan Kakkar, MBBS, MD (Gold Medal)
22. Which of the following functions is carried out by cytoskeleton in a cell?
(1) Protein synthesis
(2) Motility
(3) Transportation
(4) Nuclear division
Answer (2)
Dr. Shivankan Kakkar, MBBS, MD (Gold Medal)
The cytoskeleton in a cell carries out several functions12
.
1.Protein Synthesis: The cytoskeleton itself does not directly participate in
protein synthesis. Protein synthesis is primarily carried out by ribosomes, which
are not part of the cytoskeleton.
2.Motility: Yes, the cytoskeleton plays a crucial role in cell motility12
.
Microfilaments, a component of the cytoskeleton, aid in cell motility and allow
single-celled organisms like amoebas to move1
.
3.Transportation: The cytoskeleton facilitates the transport of molecules within
the cell12
. Microfilaments are involved in cytoplasmic streaming, which is the
flowing of cytosol (the liquid part of the cytoplasm) throughout the cell,
transporting nutrients and cell organelles1
.
4.Nuclear Division: While the cytoskeleton does play a role in cell division, it
does not directly carry out nuclear division. However, microtubules, another
component of the cytoskeleton, help in dividing chromosomes during cell
division2
.
Dr. Shivankan Kakkar, MBBS, MD (Gold Medal)
23. Once the undigested and unabsorbed substances enter the caecum, their
backflow is prevented by
(1) Ileo-caecal valve
(2) Gastro-oesophageal sphincter
(3) Pyloric sphincter
(4) Sphincter of Oddi
Answer (1)
Dr. Shivankan Kakkar, MBBS, MD (Gold Medal)
The Ileo-caecal valve (1) is a sphincter muscle valve that separates the small
intestine and the large intestine. Its critical function is to limit the reflux of colonic
contents into the ileum1
. This valve is located at the junction between the ileum and
the cecum1
.
So, the correct answer is (1) Ileo-caecal valve.
Dr. Shivankan Kakkar, MBBS, MD (Gold Medal)
24. Match List I with List II.
List I List II
A. Vasectomy I. Oral method
B. Coitus interruptus II. Barrier method
C. Cervical caps III. Surgical method
D. Saheli IV. Natural method
Choose the correct answer from the options given below:
(1) A-III, B-IV, C-II, D-I
(2) A-II, B-III, C-I, D-IV
(3) A-IV, B-II, C-I, D-III
(4) A-III, B-I, C-IV, D-II
Answer (1)
Dr. Shivankan Kakkar, MBBS, MD (Gold Medal)
• Vasectomy (A): This is a surgical procedure that makes a person who can
produce sperm unable to cause a pregnancy1
. It is a surgical method (III)1
.
• Coitus interruptus (B): Also known as the withdrawal or “pull out” method, it
involves withdrawing the penis from the vagina before ejaculation. It is
considered a natural method (IV)1
.
• Cervical caps ©: These are small, thimble-shaped cups made of silicone. They
are inserted into the vagina before sex to prevent pregnancy. They are
considered a barrier method (II)1
.
• Saheli (D): This is a non-hormonal oral contraceptive pill, which is taken once
a week. It is an oral method (I)1
.
So, the correct answer is indeed (1) A-III, B-IV, C-II, D-I.
Dr. Shivankan Kakkar, MBBS, MD (Gold Medal)
25. Match List I with List II.
List I (Type of Joint) List II (Found between)
A. Cartilaginous Joint I. Between flat skull bones
B. Ball and Socket Joint II. Between adjacent vertebrae in vertebral column
C. Fibrous Joint III. Between carpal and metacarpal of thumb
D. Saddle Joint IV. Between Humerus and Pectoral girdle
Choose the correct answer from the options given below:
(1) A-II, B-IV, C-I, D-III
(2) A-I, B-IV, C-III, D-II
(3) A-II, B-IV, C-III, D-I
(4) A-III, B-I, C-II, D-IV
Answer (1)
Dr. Shivankan Kakkar, MBBS, MD (Gold Medal)
• Cartilaginous Joint (A): These are partly movable joints comprising of
symphysis or synchondrosis joints. These joints occur only in those regions
where the connection between the articulating bones is made up of cartilage1
.
They are found between adjacent vertebrae in the vertebral column (II)1
.
• Ball and Socket Joint (B): This type of joint allows for the most movement,
including rotation. An example of a ball and socket joint is the shoulder joint,
which is between the Humerus and the Pectoral girdle (IV)1
.
• Fibrous Joint ©: These are joints in which bones are joined by dense
connective tissue that is rich in collagen fibers. These joints are also called
sutures. The joints between flat skull bones (I) are fibrous joints1
.
• Saddle Joint (D): This type of joint allows movement in two directions, like
forward and backward or side to side. An example of a saddle joint is the one
between the carpal and metacarpal of the thumb (III)1
.
So, the correct answer is (1) A-II, B-IV, C-I, D-III.
Dr. Shivankan Kakkar, MBBS, MD (Gold Medal)
26. Given below are two statements:
Statement I: RNA mutates at a faster rate.
Statement II: Viruses having RNA genome and shorter life span mutate and evolve
faster.
In the light of the above statements, choose the correct answer from the options
given below:
(1) Both Statement I and Statement II are false.
(2) Statement I is true but Statement II is false.
(3) Statement I is false but Statement II is true.
(4) Both Statement I and Statement II are true.
Answer (4)
Dr. Shivankan Kakkar, MBBS, MD (Gold Medal)
• Statement I: This statement is true. RNA mutates at a faster rate than DNA
because RNA is more prone to errors during replication, repair, and
recombination. RNA polymerases, the enzymes that copy RNA, lack
proofreading mechanisms and have higher error rates than DNA polymerases1
.
RNA is also more chemically unstable and reactive than DNA, and can be
damaged by various environmental factors1
.
• Statement II: This statement is true. Viruses having RNA genome and shorter
life span mutate and evolve faster than viruses having DNA genome and longer
life span. This is because RNA viruses have higher mutation rates per
replication cycle, and shorter life span means more replication cycles per unit of
time. Therefore, RNA viruses accumulate more mutations and genetic diversity
in a given population, which increases their chances of adaptation and
evolution1
.
Therefore, the correct answer is (4) Both Statement I and Statement II are true.
Dr. Shivankan Kakkar, MBBS, MD (Gold Medal)
27. Given below are two statements:
Statement I: In prokaryotes, the positively charged DNA is held with some
negatively charged proteins in a region called nucleoid.
Statement II: In eukaryotes, the negatively charged DNA is wrapped around the
positively charged histone octamer to form nucleosome.
In the light of the above statements, choose the correct answer from the options
given below:
(1) Both Statement I and Statement II are false.
(2) Statement I is correct but Statement II is false.
(3) Statement I is incorrect but Statement II is true.
(4) Both Statement I and Statement II are true.
Answer (3)
Dr. Shivankan Kakkar, MBBS, MD (Gold Medal)
DNA Packaging: Nucleosomes and Chromatin
• Statement I: This statement is incorrect. In prokaryotes, the negatively charged
DNA is held with some positively charged proteins in a region called the
nucleoid. The DNA in prokaryotes is double-stranded and circular, and it is not
associated with histones, but with other types of basic proteins that help
compact and organize it12345
.
• Statement II: This statement is true. In eukaryotes, the negatively charged
DNA is wrapped around the positively charged histone octamer to form a
nucleosome. A nucleosome is the basic unit of DNA packaging in eukaryotes,
and it consists of 146 base pairs of DNA wrapped around a core of eight histone
proteins (two copies each of H2A, H2B, H3, and H4)67
. Nucleosomes help
condense the DNA into a smaller volume and regulate its accessibility for
transcription and replication67
.
Therefore, the correct answer is (3) Statement I is incorrect but Statement II is true.
Dr. Shivankan Kakkar, MBBS, MD (Gold Medal)
28. Which one of the following symbols represents mating between relatives in
human pedigree analysis?
Answer (1)
Dr. Shivankan Kakkar, MBBS, MD (Gold Medal)
Mating between relatives, also known as consanguineous mating, is the mating of
individuals who are related by blood or common ancestry1
. Consanguineous mating
can increase the risk of genetic disorders, as it reduces the genetic diversity and
increases the chances of inheriting recessive alleles12
.
In human pedigree analysis, consanguineous mating is represented by a double
horizontal line between the symbols of the mating partners3
.
Dr. Shivankan Kakkar, MBBS, MD (Gold Medal)
29. Select the correct group/set of Australian Marsupials exhibiting adaptive
radiation.
(1) Numbat, Spotted cuscus, Flying phalanger
(2) Mole, Flying squirrel, Tasmanian tiger cat
(3) Lemur, Anteater, Wolf
(4) Tasmanian wolf, Bobcat, Marsupial mole
Answer (1)
Dr. Shivankan Kakkar, MBBS, MD (Gold Medal)
Australian marsupials such as Numbat, Spotted cuscus, and Flying phalanger are
examples of adaptive radiation, which is the diversification of a single ancestral
species into a variety of different forms to exploit different ecological niches. The
other options are not correct because they either include placental mammals or non-
Australian animals.
Dr. Shivankan Kakkar, MBBS, MD (Gold Medal)
30. Which of the following is not a cloning vector?
(1) YAC
(2) pBR322
(3) Probe
(4) BAC
Answer (3)
Dr. Shivankan Kakkar, MBBS, MD (Gold Medal)
A probe is not a cloning vector because it is a short DNA or RNA fragment that is
used to detect the presence or location of a specific sequence in a sample. A cloning
vector is a small piece of DNA that can be stably maintained in an organism, and
into which a foreign DNA fragment can be inserted for cloning purposes1
This link, explains the definition, features and types of cloning vectors, such as
plasmids, bacteriophages, phagemids and more.
Dr. Shivankan Kakkar, MBBS, MD (Gold Medal)
31. Match List I with List II.
List I List II
A. Heroin I. Effect on cardiovascular system
B. Marijuana II. Slow down body function
C. Cocaine III. Painkiller
D. Morphine IV. Interfere with transport of dopamine
Choose the correct answer from the options given below:
(1) A-I, B-II, C-III, D-IV
(2) A-IV, B-III, C-II, D-I
(3) A-III, B-IV, C-I, D-II
(4) A-II, B-I, C-IV, D-III
Answer (4)
Dr. Shivankan Kakkar, MBBS, MD (Gold Medal)
• Heroin (A): Heroin is an opioid that is often used illegally for its euphoric
effects. It is known to slow down body functions, which corresponds to II12345
.
• Marijuana (B): Marijuana is a psychoactive drug from the Cannabis plant used
for medical or recreational purposes. It has various effects on the body,
including effects on the cardiovascular system, which corresponds to I678910
.
• Cocaine ©: Cocaine is a strong stimulant most frequently used as a recreational
drug. It is known for its effect of interfering with the transport of dopamine, a
neurotransmitter associated with pleasure and reward, which corresponds to
IV111213
.
• Morphine (D): Morphine is a pain medication of the opiate variety. It is used to
help relieve severe or acute or chronic pain, which corresponds to III2
.
The correct matching is A-II, B-I, C-IV, D-III, which is option (4).
Dr. Shivankan Kakkar, MBBS, MD (Gold Medal)
32. Which one of the following techniques does not serve the purpose of early
diagnosis of a disease for its early treatment?
(1) Serum and Urine analysis
(2) Polymerase Chain Reaction (PCR) technique
(3) Enzyme Linked Immuno-Sorbent Assay (ELISA) technique
(4) Recombinant DNA Technology
Answer (4)
Dr. Shivankan Kakkar, MBBS, MD (Gold Medal)
• Serum and Urine analysis: These are conventional diagnostic methods used to
detect biomarkers for various diseases. For instance, serum and urine tests can
provide early detection of chronic kidney disease1
.
• Polymerase Chain Reaction (PCR) technique: PCR is a molecular biology
technique that amplifies a small amount of DNA, enabling analysis to be
conducted much more easily. It is widely used in medical and biological
research labs for the diagnosis of infectious diseases, genetic disorders, and
cancers345
.
• Enzyme Linked Immuno-Sorbent Assay (ELISA) technique: ELISA is an
immunological assay used to measure antibodies, antigens, proteins, and
glycoproteins in biological samples. It is commonly used for the diagnosis of
various diseases, including HIV infection, pregnancy tests, and measurement of
cytokines or soluble receptors in cell supernatant or serum6789
.
• Recombinant DNA Technology: This technology involves the introduction of
a foreign piece of DNA structure into the genome which contains our gene of
interest. It has been used for producing artificial DNA to generate the desired
product10
.
Among these techniques, Recombinant DNA Technology (option 4) is the one that
does not primarily serve the purpose of early diagnosis of a disease for its early
treatment.
Dr. Shivankan Kakkar, MBBS, MD (Gold Medal)
33. Given below are two statements: one is labelled as Assertion A and the other is
labelled as Reason R.
Assertion A: Endometrium is necessary for implantation of blastocyst.
Reason R: In the absence of fertilization, the corpus luteum degenerates that causes
disintegration of endometrium.
In the light of the above statements, choose the correct answer from the options
given below:
(1) Both A and R are true but R is NOT the correct explanation of A.
(2) A is true but R is false.
(3) A is false but R is true.
(4) Both A and R are true and R is the correct explanation of A.
Answer (1)
Dr. Shivankan Kakkar, MBBS, MD (Gold Medal)
• Assertion A: The endometrium, which is the inner lining of the uterus, is
indeed necessary for the implantation of a blastocyst1234
. A blastocyst is a ball
of cells that forms early in a pregnancy, about five to six days after a sperm
fertilizes an egg. It implants in the uterine wall, eventually becoming the
embryo and then the fetus1
. If the blastocyst doesn’t implant in the
endometrium, pregnancy will not occur15
.
• Reason R: In the absence of fertilization, the corpus luteum, a temporary
endocrine structure, does degenerate678910
. This leads to a decline in the
progesterone level, which causes disintegration of the endometrium leading to
menstruation678910
. However, this is not the reason why the endometrium is
necessary for the implantation of a blastocyst.
So, both A and R are true, but R is NOT the correct explanation of A. Therefore, the
correct answer is option (1).
Dr. Shivankan Kakkar, MBBS, MD (Gold Medal)
34. Match List I with List II.
List I List II
A. Gene ‘a’ I. ß-galactosidase
B. Gene ‘y’ II. Transacetylase
C. Gene ‘i’ III. Permease
D. Gene ‘z’ IV. Repressor protein
Choose the correct answer from the options given below:
(1) A-II, B-III, C-IV, D-I
(2) A-III, B-IV, C-I, D-II
(3) A-III, B-I, C-IV, D-II
(4) A-II, B-I, C-IV, D-III
Answer (1)
Dr. Shivankan Kakkar, MBBS, MD (Gold Medal)
• Gene ‘a’: This gene codes for a transacetylase12
. Transacetylase is an enzyme
that transfers an acetyl group from one molecule to another. In the context of
the lac operon in E.coli, the ‘a’ gene encodes a transacetylase that transfers an
acetyl group from acetyl-CoA to β-galactosides12
. Therefore, Gene ‘a’
corresponds to II. Transacetylase.
• Gene ‘y’: This gene codes for permease345
. Permease is a type of protein that
aids in the transport of a specific type of molecule across the cell membrane. In
the lac operon, the ‘y’ gene codes for β-galactoside permease, a protein that
enables the transport of lactose into the cell345
. Therefore, Gene ‘y’ corresponds
to III. Permease.
• Gene ‘i’: This gene codes for the repressor protein678
. The repressor protein
binds to the operator region of the operon and prevents transcription of the
downstream genes. In the lac operon, the ‘i’ gene codes for the lac repressor,
which binds to the operator region of the lac operon and prevents transcription
of the ‘z’, ‘y’, and ‘a’ genes in the absence of lactose678
. Therefore, Gene ‘i’
corresponds to IV. Repressor protein.
• Gene ‘z’: This gene codes for β-galactosidase91011
. β-galactosidase is an
enzyme that catalyzes the hydrolysis of β-galactosides into monosaccharides. In
the lac operon, the ‘z’ gene codes for β-galactosidase, which catalyzes the
hydrolysis of lactose into glucose and galactose91011
. Therefore, Gene ‘z’
corresponds to I. ß-galactosidase.
The correct matching is A-II, B-III, C-IV, D-I, which corresponds to option (1).
Dr. Shivankan Kakkar, MBBS, MD (Gold Medal)
35. Given below are two statements: one is labelled as Assertion A and the other is
labelled as Reason R.
Assertion A: Nephrons are of two types: Cortical & Juxta medullary, based on their
relative position in cortex and medulla.
Reason R: Juxta medullary nephrons have short loop of Henle whereas, cortical
nephrons have longer loop of Henle.
In the light of the above statements, choose the correct answer from the options
given below:
(1) Both A and R are true but R is NOT the correct explanation of A.
(2) A is true but R is false.
(3) A is false but R is true.
(4) Both A and R are true and R is the correct explanation of A.
Answer (2)
Dr. Shivankan Kakkar, MBBS, MD (Gold Medal)
• Assertion A: Nephrons are indeed of two types: Cortical and Juxta medullary,
based on their relative position in the cortex and medulla1234
. Cortical nephrons
are located primarily in the renal cortex and make up about 85% of nephrons in
the human kidney. Juxtamedullary nephrons have their renal corpuscles close to
the medulla, and their loops of Henle extend deep into the renal medulla256
.
• Reason R: This statement is incorrect. In fact, it’s the opposite: Juxtamedullary
nephrons have a longer loop of Henle which extends deeper into the inner
medulla, while cortical nephrons contain a shorter loop of Henle which only
extends into the outer region of the renal medulla25789
.
A is true but R is false. Therefore, the correct answer is option (2).
Dr. Shivankan Kakkar, MBBS, MD (Gold Medal)
SECTION B
36. Which of the following is characteristic feature of cockroach regarding sexual
dimorphism?
(1) Presence of anal styles
(2) Presence of sclerites
(3) Presence of anal cerci
(4) Dark brown body colour and anal cerci
Answer (1)
Dr. Shivankan Kakkar, MBBS, MD (Gold Medal)
1.Presence of anal styles: Anal styles are present in male cockroaches and absent
in females1234
. They are small, thread-like structures that are used during
mating3
. The male uses them to grasp the female during copulation and they
also play a role in transferring sperm3
.
2.Presence of sclerites: Sclerites are chitinous exoskeleton plates that provide
rigidity and protection567
. They are present in all the body segments of both
male and female cockroaches56
.
3.Presence of anal cerci: Anal cerci are a pair of filamentous structures that are
present in both male and female cockroaches82910
. They are sensory structures
that help the cockroach to detect vibrations and other stimuli in their
environment910
.
4.Dark brown body colour and anal cerci: The body of the cockroach is
covered by a hard chitinous exoskeleton that is brown in colour11
. Both male
and female cockroaches have anal cerci82910
.
The correct answer is option (1) - Presence of anal styles, as this is the only feature
that is sexually dimorphic, i.e., different between male and female cockroaches1234
.
Dr. Shivankan Kakkar, MBBS, MD (Gold Medal)
37. Select the correct statements.
A. Tetrad formation is seen during Leptotene.
B. During Anaphase, the centromeres split and chromatids separate.
C. Terminalization takes place during Pachytene.
D. Nucleolus, Golgi complex and ER are reformed during Telophase.
E. Crossing over takes place between sister chromatids of homologous
chromosome.
Choose the correct answer from the options given below:
(1) B and D only
(2) A,C and E only
(3) B and E only
(4) A and C only
Answer (1)
Dr. Shivankan Kakkar, MBBS, MD (Gold Medal)
• Statement A: Tetrad formation is seen during Leptotene. This statement is
incorrect. Tetrad formation, also known as synapsis, occurs during the Zygotene
stage of Prophase I of meiosis1234
.
• Statement B: During Anaphase, the centromeres split and chromatids separate.
This statement is correct. During Anaphase, the centromeres of each
chromosome split, and the sister chromatids separate and move to opposite
poles56789
.
• Statement C: Terminalization takes place during Pachytene. This statement is
incorrect. Terminalization, which refers to the movement of chiasmata (sites of
crossing over) towards the ends (terminals) of the homologous chromosomes,
occurs during Diplotene, not Pachytene10113
.
• Statement D: Nucleolus, Golgi complex and ER are reformed during
Telophase. This statement is correct. During Telophase, the nuclear envelope
reforms around each set of separated chromosomes. The nucleolus, Golgi
complex, and endoplasmic reticulum (ER) are also reformed within the newly
formed nuclei121341415
.
• Statement E: Crossing over takes place between sister chromatids of
homologous chromosome. This statement is incorrect. Crossing over is the
exchange of genetic material between non-sister chromatids of homologous
chromosomes during meiosis, which results in new allelic combinations in the
daughter cells16171819
.
The correct statements are B and D only, which corresponds to option (1).
Dr. Shivankan Kakkar, MBBS, MD (Gold Medal)
38. Which of the following statements are correct?
A. An excessive loss of body fluid from the body switches off osmoreceptors.
B. ADH facilitates water reabsorption to prevent diuresis.
C. ANF causes vasodilation.
D. ADH causes increase in blood pressure.
E. ADH is responsible for decrease in GFR.
Choose the correct answer from the options given below:
(1) B, Cand D only
(2) A, B and E only
(3) C, D and E only
(4) A and B only
Answer (1)
Dr. Shivankan Kakkar, MBBS, MD (Gold Medal)
• Statement A: An excessive loss of body fluid from the body switches off
osmoreceptors. This statement is incorrect. Osmoreceptors are specialized cells
that detect changes in osmotic pressure and regulate the release of antidiuretic
hormone (ADH). Excessive loss of body fluid would lead to increased osmotic
pressure and activate osmoreceptors, not switch them off1234
.
• Statement B: ADH facilitates water reabsorption to prevent diuresis. This
statement is correct. Antidiuretic hormone (ADH), also known as vasopressin,
acts on the kidneys to increase water reabsorption. It helps prevent excessive
water loss and promotes water retention in the body56718
.
• Statement C: ANF causes vasodilation. This statement is correct. Atrial
natriuretic peptide (ANF) is a hormone released by the heart in response to
increased blood volume and pressure. It promotes vasodilation, which leads to
the relaxation of blood vessels and increased excretion of sodium and
water91011
.
• Statement D: ADH causes increase in blood pressure. This statement is correct.
ADH acts on the blood vessels, causing vasoconstriction. This constriction
increases peripheral vascular resistance, which can contribute to an increase in
blood pressure6121314
.
• Statement E: ADH is responsible for decrease in GFR. This statement is
incorrect. ADH does not directly influence the glomerular filtration rate (GFR).
It primarily affects water reabsorption in the renal tubules11516
.
The correct statements are B, C, and D only, which corresponds to option (1).
Dr. Shivankan Kakkar, MBBS, MD (Gold Medal)
39. Given below are two statements:
Statement I : During G0 phase of cell cycle, the cell is metabolically inactive.
Statement II : The centrosome undergoes duplication during S phase of interphase.
In the light of the above statements, choose the most appropriate answer from the
options given below:
(1) Both Statement I and Statement II are incorrect.
(2) Statement I is correct but Statement II is incorrect.
(3) Statement I is incorrect but Statement II is correct.
(4) Both Statement I and Statement II are correct
Answer (3)
Dr. Shivankan Kakkar, MBBS, MD (Gold Medal)
• Statement I: During G0 phase of cell cycle, the cell is metabolically inactive.
This statement is incorrect. While the G0 phase is considered a quiescent phase
where cells are not actively dividing, many cells in G0 are still metabolically
active and can carry out essential functions12
.
• Statement II: The centrosome undergoes duplication during S phase of
interphase. This statement is correct. The centrosome starts duplicating just
before, or at, the onset of S phase and the two newly formed centrosomes
participate in the assembly and organization of the mitotic spindle345
.
So, Statement I is incorrect but Statement II is correct. Therefore, the correct answer
is option (3).
Dr. Shivankan Kakkar, MBBS, MD (Gold Medal)
40. Which one of the following is NOT an advantage of inbreeding?
(1) It exposes harmful recessive genes but are eliminated by selection.
(2) Elimination of less desirable genes and accumulation of superior genes takes
place due to it.
(3) It decreases the productivity of inbred population, after continuous inbreeding.
(4) It decreases homozygosity.
Answer (3)
Dr. Shivankan Kakkar, MBBS, MD (Gold Medal)
• It exposes harmful recessive genes but are eliminated by selection.
Inbreeding can lead to the exposure of harmful recessive genes. These harmful
genes can be eliminated over time through the process of natural selection12
.
• Elimination of less desirable genes and accumulation of superior genes
takes place due to it. Inbreeding can lead to the elimination of less desirable
genes and the accumulation of superior genes. This happens as individuals with
similar genetic backgrounds mate, reducing the frequency of less desirable
genes and concentrating desirable genes3456
.
• It decreases the productivity of inbred population, after continuous
inbreeding. Continuous inbreeding can lead to a decrease in the productivity of
the inbred population. This is due to the increased expression of harmful
recessive genes, a phenomenon known as inbreeding depression7891011
. .
Continuous inbreeding can decrease the productivity of the inbred
population, which is a disadvantage, not an advantage.
• It decreases homozygosity. This statement is incorrect. Inbreeding actually
increases homozygosity, not decreases it. Inbreeding leads to an increase in
homozygosity because it involves mating between closely related
individuals781213
.
The answer is (3) because it is not an advantage of inbreeding.
Dr. Shivankan Kakkar, MBBS, MD (Gold Medal)
41. Match List I with List II.
List I List II
A. Logistic growth I. Unlimited resource availability condition
B. Exponential growth II. Limited resource availability condition
C. Expanding age pyramid III. The percent individuals of pre-reproductive age is
largest followed by reproductive and post reproductive
age groups
D. Stable age pyramid IV.The percent individuals of pre-reproductives and
reproductive age group are same
Choose the correct answer from the options given below:
(1) A-II, B-III, C-I, D-IV
(2) A-II, B-IV, C-I, D-III
(3) A-II, B-IV, C-III, D-I
(4) A-II, B-I, C-III, D-IV
Answer (4)
Dr. Shivankan Kakkar, MBBS, MD (Gold Medal)
• A. Logistic growth matches with II. Limited resource availability condition.
Logistic growth occurs when resources are limited, causing the population
growth rate to decrease as the population size approaches the environment’s
carrying capacity1234
.
• B. Exponential growth matches with I. Unlimited resource availability
condition. Exponential growth occurs when resources are abundant, allowing
the population to grow rapidly567
.
• C. Expanding age pyramid matches with III. The percent individuals of pre-
reproductive age is largest followed by reproductive and post reproductive age
groups. An expanding age pyramid is characterized by a larger percentage of
individuals in younger age groups, indicating high fertility rates and lower life
expectancies89101112
.
• D. Stable age pyramid matches with IV. The percent individuals of pre-
reproductives and reproductive age group are same. A stable age pyramid
shows a somewhat equal proportion of the population in each age group,
indicating a stable population13141516
.
The answer is (4) because it correctly matches each item in List I with the
corresponding item in List II.
Dr. Shivankan Kakkar, MBBS, MD (Gold Medal)
42. Which of the following are NOT under the control of thyroid hormone?
A. Maintenance of water and electrolyte balance
B. Regulation of basal metabolic rate
C. Normal rhythm of sleep-wake cycle
D. Development of immune system
E. Support the process of RBCs formation
Choose the correct answer from the options given below:
(1) B and C only
(2) C and D only
(3) D and E only
(4) A and D only
Answer (2)
Dr. Shivankan Kakkar, MBBS, MD (Gold Medal)
A. Maintenance of water and electrolyte balance: This is controlled by the thyroid
hormone12
.
B. Regulation of basal metabolic rate: The thyroid hormone plays a crucial role in
regulating the body’s metabolism12
.
C. Normal rhythm of sleep-wake cycle: While the thyroid hormone can influence
energy levels, it does not directly control the sleep-wake cycle12
.
D. Development of immune system: The thyroid hormone does not directly control
the development of the immune system12
.
E. Support the process of RBCs formation: The thyroid hormone does play a role in
the differentiation and development of cells2
, which could influence RBC
formation.
So, the options C and D (the sleep-wake cycle and the development of the immune
system) are not directly under the control of the thyroid hormone, which
corresponds to answer (2). Therefore, the given answer is correct.
Dr. Shivankan Kakkar, MBBS, MD (Gold Medal)
43. The unique mammalian characteristics are:
(1) hairs, pinna and mammary glands
(2) hairs, pinna and indirect development
(3) pinna, monocondylic skull and mammary glands
(4) hairs, tympanic membrane and mammary glands
Answer (1)
Dr. Shivankan Kakkar, MBBS, MD (Gold Medal)
1.Hairs, pinna, and mammary glands: All mammals have hair at some stage of
their life cycle1
. The pinna is the visible part of the ear that resides outside of
the head, and it’s a feature that most mammals have. Mammary glands, which
produce milk to feed young, are a defining characteristic of mammals1
.
2.Hairs, pinna, and indirect development: While hairs and pinna are mammalian
characteristics, indirect development is not a unique mammalian characteristic.
Many animals, not just mammals, undergo indirect development.
3.Pinna, monocondylic skull, and mammary glands: While pinna and mammary
glands are mammalian characteristics, a monocondylic skull (a skull with one
occipital condyle) is not a unique characteristic of mammals. In fact, mammals
typically have a dicondylic skull (a skull with two occipital condyles)1
.
4.Hairs, tympanic membrane, and mammary glands: While hairs and mammary
glands are mammalian characteristics, the presence of a tympanic membrane
(eardrum) is not unique to mammals. Many other animals, including birds and
reptiles, also have tympanic membranes.
So, the only option that lists exclusively mammalian characteristics is option (1):
hairs, pinna, and mammary glands. Therefore, the given answer is correct.
Dr. Shivankan Kakkar, MBBS, MD (Gold Medal)
44. Which of the following statements are correct regarding skeletal muscle?
A. Muscle bundles are held together by collagenous connective tissue layer called
fascicle.
B. Sarcoplasmic reticulum of muscle fibre is a store house of calcium ions.
C. Striated appearance of skeletal muscle fibre is due to distribution pattern of actin
and myosin proteins.
D. M line is considered as functional unit of contraction called sarcomere.
Choose the most appropriate answer from the options given below:
(1) B and C only
(2) A, C and D only
(3) C and D only
(4) A, B and Conly
Answer (1)
Dr. Shivankan Kakkar, MBBS, MD (Gold Medal)
• Statement A is incorrect because the fascicle is not the connective tissue layer,
but the bundle of muscle fibres. The connective tissue layer that surrounds the
fascicle is called the perimysium.
• Statement B is correct because the sarcoplasmic reticulum is a specialized
endoplasmic reticulum that stores and releases calcium ions during muscle
contraction and relaxation.
• Statement C is correct because the striated appearance of skeletal muscle fibre
is due to the alternating bands of thin filaments (made of actin) and thick
filaments (made of myosin) that form the myofibrils.
• Statement D is correct because the M line is the middle point of a sarcomere,
which is the functional unit of contraction in skeletal muscle. The sarcomere is
the region between two Z lines, which mark the boundaries of a myofibril
segment.
Dr. Shivankan Kakkar, MBBS, MD (Gold Medal)
45. Which of the following statements are correct?
A. Basophils are most abundant cells of the total WBCs
B. Basophils secrete histamine, serotonin and heparin
C. Basophils are involved in inflammatory response
D. Basophils have kidney shaped nucleus
E. Basophils are agranulocytes
Choose the correct answer from the options given below:
(1) C and E only
(2) B and C only
(3) A and B only
(4) D and E only
Answer (2)
Dr. Shivankan Kakkar, MBBS, MD (Gold Medal)
The correct answer is (2) B and C only. Here is an explanation:
• Statement A is incorrect because basophils are the least abundant cells of the
total WBCs, accounting for only about 0.5% of the circulating leukocytes12
.
• Statement B is correct because basophils secrete histamine, which is a
vasodilator that increases blood flow to the tissues, and heparin, which is an
anticoagulant that prevents blood clotting12
. Basophils also secrete serotonin,
which is a neurotransmitter that modulates pain and mood3
.
• Statement C is correct because basophils are involved in inflammatory
response, especially in allergic reactions and parasitic infections12
. Basophils
release histamine and other mediators that cause symptoms such as itching,
swelling, and sneezing12
.
• Statement D is incorrect because basophils have a lobed nucleus, not a kidney-
shaped nucleus12
. The nucleus of basophils is often obscured by the large
cytoplasmic granules that stain dark purple or black12
.
• Statement E is incorrect because basophils are granulocytes, not
agranulocytes12
. Granulocytes are a type of white blood cells that have granules
in their cytoplasm, whereas agranulocytes are a type of white blood cells that
lack granules in their cytoplasm12
.
Dr. Shivankan Kakkar, MBBS, MD (Gold Medal)
46. Which one of the following is the sequence on corresponding coding strand, if
the sequence on mRNA formed is as follows
5’AUCGAUCGAUCGAUCGAUCGAUCG AUCG 3’?
(1) 3’ UAGCUAGCUAGCUAGCUAGCUAGCUAGC 5’
(2) 5’ ATCGATCGATCGATCGATCGATCGATCG 3’
(3) 3’ ATCGATCGATCGATCGATCGATCGATCG 5’
(4) 5’ UAGCUAGCUAGCUAGCUAGCUAGCUAGC 3’
Answer (2)
Dr. Shivankan Kakkar, MBBS, MD (Gold Medal)
The correct answer is (2) 5’ ATCGATCGATCGATCGATCGATCGATCG 3’.
Here is an explanation:
• The coding strand is the DNA strand whose base sequence is identical to the
base sequence of the mRNA transcript produced, except for the replacement of
thymine (T) in DNA with uracil (U) in mRNA12
.
• The coding strand is also presented in the 5’ to 3’ direction, which is the same
direction as the mRNA transcript12
.
• Therefore, to find the sequence of the coding strand, we just need to replace U
with T in the mRNA sequence, and keep the same direction.
• The mRNA sequence is 5’AUCGAUCGAUCGAUCGAUCGAUCG AUCG 3’,
so the coding strand sequence is 5’ATCGATCGATCGATCGATCGATCG
ATCG 3’.
Dr. Shivankan Kakkar, MBBS, MD (Gold Medal)
47. The parts of human brain that helps in regulation of sexual behaviour,
expression of excitement, pleasure, rage, fear etc. are:
(1) Corpora quadrigemina and hippocampus
(2) Brain stem and epithalamus
(3) Corpus callosum and thalamus
(4) Limbic system and hypothalamus
Answer (4)
Dr. Shivankan Kakkar, MBBS, MD (Gold Medal)
The correct answer is (4) Limbic system and hypothalamus. Here is an
explanation:
• The limbic system is a complex network of brain structures that includes the
amygdala, hippocampus, cingulate gyrus, and other parts12
. The limbic system
is involved in regulating emotions, motivation, memory, and learning12
. It also
plays a key role in sexual behaviour, as it mediates the reward and pleasure
associated with mating34
.
• The hypothalamus is a small but important region of the brain that lies below
the thalamus and above the pituitary gland12
. The hypothalamus is responsible
for maintaining the body’s homeostasis, or internal balance, by regulating
various physiological functions such as temperature, hunger, thirst, sleep, and
circadian rhythms12
. The hypothalamus also controls the secretion of hormones
from the pituitary gland, which in turn affect the reproductive system, growth,
metabolism, and stress response12
. The hypothalamus also influences sexual
behaviour, as it integrates sensory and hormonal signals and coordinates the
sexual arousal and orgasm34
.
Dr. Shivankan Kakkar, MBBS, MD (Gold Medal)
48. Match List I with List II.
List I List II
A. Mast cells I. Ciliated epithelium
B. Inner surface of bronchiole II. Areolar connective tissue
C. Blood III. Cuboidal epithelium
D. Tubular parts of nephron IV. Specialised connective tissue
Choose the correct answer from the options give below:
(1) A-II, B-III, C-I, D-IV
(2) A-II, B-I, C-IV, D-III
(3) A-III, B-IV, C-II, D-I
(4) A-I, B-II, C-IV, D-III
Answer (2)
Dr. Shivankan Kakkar, MBBS, MD (Gold Medal)
• A. Mast cells are a type of immune cell that are found in connective tissue,
especially in the skin, respiratory tract, and gastrointestinal tract. They release
histamine and other mediators that cause allergic reactions and inflammation.
Mast cells are found in areolar connective tissue, which is a loose type of
connective tissue that fills the spaces between organs and tissues12
.
• B. The inner surface of bronchiole is lined by ciliated epithelium, which is a
type of epithelial tissue that has hair-like projections called cilia on its surface.
The cilia help to move mucus and trapped particles out of the respiratory tract34
.
• C. Blood is a specialized connective tissue that consists of plasma, blood cells,
and platelets. It circulates throughout the body and transports oxygen, nutrients,
hormones, and waste products. Blood is classified as a connective tissue
because it originates from the mesoderm and has a matrix (plasma) that
supports the cells56
.
• D. The tubular parts of nephron are the structures in the kidney that filter the
blood and produce urine. They include the proximal tubule, the loop of Henle,
the distal tubule, and the collecting duct. The tubular parts of nephron are lined
by cuboidal epithelium, which is a type of epithelial tissue that has cube-shaped
cells. The cuboidal epithelium facilitates the reabsorption and secretion of water
and solutes in the nephron78
.
Dr. Shivankan Kakkar, MBBS, MD (Gold Medal)
49. In cockroach, excretion is brought about by-
A. Phallic gland
B. Uricose gland
C. Nephrocytes
D. Fat body
E. Collaterial glands
(1) A, B and E only
(2) B, Cand D only
(3) B and D only
(4) A and E only
Answer (2)
Dr. Shivankan Kakkar, MBBS, MD (Gold Medal)
• A. Phallic gland: This is a gland that is associated with the male reproductive
system of cockroaches. It secretes a substance that forms the spermatophore,
which is a capsule that contains the sperm. The phallic gland is not involved in
excretion1
.
• B. Uricose gland: This is a gland that is located in the hindgut of cockroaches. It
secretes uric acid, which is the main nitrogenous waste product of cockroaches.
The uricose gland helps to conserve water and maintain osmotic balance. The
uricose gland is involved in excretion23
.
• C. Nephrocytes: These are cells that are found in the haemocoel of cockroaches.
They are phagocytic cells that engulf and store waste materials, such as uric
acid crystals, from the haemolymph. They also help to regulate the pH and ion
concentration of the haemolymph. Nephrocytes are involved in excretion24
.
• D. Fat body: This is a tissue that is distributed throughout the body cavity of
cockroaches. It consists of various types of cells, such as trophocytes,
mycetocytes, and urocytes. The fat body performs several functions, such as
storage of nutrients, synthesis of proteins and lipids, and detoxification of
toxins. The fat body also contains urocytes, which are cells that store uric acid
in vacuoles. The fat body is involved in excretion2
.
• E. Collaterial glands: These are glands that are associated with the female
reproductive system of cockroaches. They secrete a substance that forms the
ootheca, which is a protective case that contains the eggs. The collaterial glands
are not involved in excretion1
.
Dr. Shivankan Kakkar, MBBS, MD (Gold Medal)
50. Select the correct statements with reference to chordates.
A. Presence of a mid-dorsal, solid and double nerve cord.
B. Presence of closed circulatory system.
C. Presence of paired pharyngeal gill slits.
D. Presence of dorsal heart
E. Triploblastic pseudocoelomate animals.
Choose the correct answer from the options given below:
(1) B and C only
(2) B,D and E only
(3) C, D and E only
(4) A, Cand D only
Answer (1)
Dr. Shivankan Kakkar, MBBS, MD (Gold Medal)
• Statement A is incorrect because chordates have a mid-dorsal, hollow and
single nerve cord, not a solid and double nerve cord12
. The nerve cord is located
above the notochord and develops into the brain and spinal cord in vertebrates12
.
• Statement B is correct because chordates have a closed circulatory system,
which means that the blood is confined within blood vessels and pumped by a
heart12
. The circulatory system transports oxygen, nutrients, hormones, and
waste products throughout the body12
.
• Statement C is correct because chordates have paired pharyngeal gill slits,
which are openings in the pharynx (throat) that connect to the external
environment12
. The gill slits function as a filter-feeding device in some
invertebrate chordates, and as a respiratory organ in aquatic vertebrates12
.
• Statement D is incorrect because chordates have a ventral heart, not a dorsal
heart12
. The heart is located below the pharynx and pumps blood through the
circulatory system12
.
• Statement E is incorrect because chordates are triploblastic coelomate animals,
not pseudocoelomate animals12
. This means that they have three germ layers
(ectoderm, mesoderm, and endoderm) and a true body cavity (coelom) that is
lined by mesoderm12
.

More Related Content

Similar to Zoology Question Bank for the NEET-UG 2024 Exam

Gnc paper solution chn ii 3rd year GNM years 2014 and 2015
Gnc paper solution chn ii 3rd year GNM  years 2014 and 2015Gnc paper solution chn ii 3rd year GNM  years 2014 and 2015
Gnc paper solution chn ii 3rd year GNM years 2014 and 2015Khima Rudach
 
Antibiotic Investigation
Antibiotic InvestigationAntibiotic Investigation
Antibiotic InvestigationSheila Guy
 
Discovering the 100 Trillion Bacteria Living Within Each of Us
Discovering the 100 Trillion Bacteria Living Within Each of UsDiscovering the 100 Trillion Bacteria Living Within Each of Us
Discovering the 100 Trillion Bacteria Living Within Each of UsLarry Smarr
 
Discovering the 100 Trillion Bacteria Living Within Each of Us
Discovering the 100 Trillion Bacteria Living Within Each of UsDiscovering the 100 Trillion Bacteria Living Within Each of Us
Discovering the 100 Trillion Bacteria Living Within Each of UsLarry Smarr
 
How Studying Astrophysics and Coral Reefs Enabled Me to Become an Empowered,...
How Studying Astrophysics and Coral Reefs Enabled Me to Become an Empowered,...How Studying Astrophysics and Coral Reefs Enabled Me to Become an Empowered,...
How Studying Astrophysics and Coral Reefs Enabled Me to Become an Empowered,...Larry Smarr
 
Session 3 2012 fmhs_paraphrasing and summarising
Session 3 2012 fmhs_paraphrasing and summarisingSession 3 2012 fmhs_paraphrasing and summarising
Session 3 2012 fmhs_paraphrasing and summarisingmtfinn
 
Research by Mahendra Kumar Trivedi - Antimicrobial Susceptibility Pattern, Bi...
Research by Mahendra Kumar Trivedi - Antimicrobial Susceptibility Pattern, Bi...Research by Mahendra Kumar Trivedi - Antimicrobial Susceptibility Pattern, Bi...
Research by Mahendra Kumar Trivedi - Antimicrobial Susceptibility Pattern, Bi...Abby Keif
 
Awareness about Intestinal Cancer in University Student_Crimson Publishers
Awareness about Intestinal Cancer in University Student_Crimson PublishersAwareness about Intestinal Cancer in University Student_Crimson Publishers
Awareness about Intestinal Cancer in University Student_Crimson PublishersCrimsonpublishersCancer
 
Evidence Based Practice_lecture 5_slides
Evidence Based Practice_lecture 5_slidesEvidence Based Practice_lecture 5_slides
Evidence Based Practice_lecture 5_slidesZakCooper1
 
Exposure to Toxic Environmental Agents
Exposure to Toxic Environmental Agents Exposure to Toxic Environmental Agents
Exposure to Toxic Environmental Agents v2zq
 
Previous year question on abortion based on neet pg, usmle, plab and fmge or ...
Previous year question on abortion based on neet pg, usmle, plab and fmge or ...Previous year question on abortion based on neet pg, usmle, plab and fmge or ...
Previous year question on abortion based on neet pg, usmle, plab and fmge or ...Abhishek Gupta
 
The Pregnancy Microbiome: The Link Between Maternal Periodontitis and Adverse...
The Pregnancy Microbiome: The Link Between Maternal Periodontitis and Adverse...The Pregnancy Microbiome: The Link Between Maternal Periodontitis and Adverse...
The Pregnancy Microbiome: The Link Between Maternal Periodontitis and Adverse...Crimsonpublishers-IGRWH
 
Impact of gut microbes on gastrointestinal disease
Impact of gut microbes on gastrointestinal diseaseImpact of gut microbes on gastrointestinal disease
Impact of gut microbes on gastrointestinal diseaseSandra Unorji
 
Rapid Performance Appraisal
Rapid Performance AppraisalRapid Performance Appraisal
Rapid Performance AppraisalSheri Elliott
 
BIODIVERSITY and HEALTHY POPULATIONSECS111 P DR. SE.docx
BIODIVERSITY and HEALTHY POPULATIONSECS111  P DR. SE.docxBIODIVERSITY and HEALTHY POPULATIONSECS111  P DR. SE.docx
BIODIVERSITY and HEALTHY POPULATIONSECS111 P DR. SE.docxtangyechloe
 
Antimicrobial Susceptibility Pattern, Biochemical Characteristics and Biotypi...
Antimicrobial Susceptibility Pattern, Biochemical Characteristics and Biotypi...Antimicrobial Susceptibility Pattern, Biochemical Characteristics and Biotypi...
Antimicrobial Susceptibility Pattern, Biochemical Characteristics and Biotypi...albertdivis
 

Similar to Zoology Question Bank for the NEET-UG 2024 Exam (19)

Gnc paper solution chn ii 3rd year GNM years 2014 and 2015
Gnc paper solution chn ii 3rd year GNM  years 2014 and 2015Gnc paper solution chn ii 3rd year GNM  years 2014 and 2015
Gnc paper solution chn ii 3rd year GNM years 2014 and 2015
 
Antibiotic Investigation
Antibiotic InvestigationAntibiotic Investigation
Antibiotic Investigation
 
Discovering the 100 Trillion Bacteria Living Within Each of Us
Discovering the 100 Trillion Bacteria Living Within Each of UsDiscovering the 100 Trillion Bacteria Living Within Each of Us
Discovering the 100 Trillion Bacteria Living Within Each of Us
 
Discovering the 100 Trillion Bacteria Living Within Each of Us
Discovering the 100 Trillion Bacteria Living Within Each of UsDiscovering the 100 Trillion Bacteria Living Within Each of Us
Discovering the 100 Trillion Bacteria Living Within Each of Us
 
How Studying Astrophysics and Coral Reefs Enabled Me to Become an Empowered,...
How Studying Astrophysics and Coral Reefs Enabled Me to Become an Empowered,...How Studying Astrophysics and Coral Reefs Enabled Me to Become an Empowered,...
How Studying Astrophysics and Coral Reefs Enabled Me to Become an Empowered,...
 
Session 3 2012 fmhs_paraphrasing and summarising
Session 3 2012 fmhs_paraphrasing and summarisingSession 3 2012 fmhs_paraphrasing and summarising
Session 3 2012 fmhs_paraphrasing and summarising
 
Unit 3
Unit 3Unit 3
Unit 3
 
Research by Mahendra Kumar Trivedi - Antimicrobial Susceptibility Pattern, Bi...
Research by Mahendra Kumar Trivedi - Antimicrobial Susceptibility Pattern, Bi...Research by Mahendra Kumar Trivedi - Antimicrobial Susceptibility Pattern, Bi...
Research by Mahendra Kumar Trivedi - Antimicrobial Susceptibility Pattern, Bi...
 
Awareness about Intestinal Cancer in University Student_Crimson Publishers
Awareness about Intestinal Cancer in University Student_Crimson PublishersAwareness about Intestinal Cancer in University Student_Crimson Publishers
Awareness about Intestinal Cancer in University Student_Crimson Publishers
 
Evidence Based Practice_lecture 5_slides
Evidence Based Practice_lecture 5_slidesEvidence Based Practice_lecture 5_slides
Evidence Based Practice_lecture 5_slides
 
Exposure to Toxic Environmental Agents
Exposure to Toxic Environmental Agents Exposure to Toxic Environmental Agents
Exposure to Toxic Environmental Agents
 
Previous year question on abortion based on neet pg, usmle, plab and fmge or ...
Previous year question on abortion based on neet pg, usmle, plab and fmge or ...Previous year question on abortion based on neet pg, usmle, plab and fmge or ...
Previous year question on abortion based on neet pg, usmle, plab and fmge or ...
 
The Pregnancy Microbiome: The Link Between Maternal Periodontitis and Adverse...
The Pregnancy Microbiome: The Link Between Maternal Periodontitis and Adverse...The Pregnancy Microbiome: The Link Between Maternal Periodontitis and Adverse...
The Pregnancy Microbiome: The Link Between Maternal Periodontitis and Adverse...
 
Impact of gut microbes on gastrointestinal disease
Impact of gut microbes on gastrointestinal diseaseImpact of gut microbes on gastrointestinal disease
Impact of gut microbes on gastrointestinal disease
 
Rapid Performance Appraisal
Rapid Performance AppraisalRapid Performance Appraisal
Rapid Performance Appraisal
 
orang esteghamat
 orang esteghamat orang esteghamat
orang esteghamat
 
Phillips Course Sheets
Phillips Course SheetsPhillips Course Sheets
Phillips Course Sheets
 
BIODIVERSITY and HEALTHY POPULATIONSECS111 P DR. SE.docx
BIODIVERSITY and HEALTHY POPULATIONSECS111  P DR. SE.docxBIODIVERSITY and HEALTHY POPULATIONSECS111  P DR. SE.docx
BIODIVERSITY and HEALTHY POPULATIONSECS111 P DR. SE.docx
 
Antimicrobial Susceptibility Pattern, Biochemical Characteristics and Biotypi...
Antimicrobial Susceptibility Pattern, Biochemical Characteristics and Biotypi...Antimicrobial Susceptibility Pattern, Biochemical Characteristics and Biotypi...
Antimicrobial Susceptibility Pattern, Biochemical Characteristics and Biotypi...
 

More from Shivankan Kakkar

STRUCTURAL ORGANIZATION IN ANIMALS (PART 1)
STRUCTURAL ORGANIZATION IN ANIMALS (PART 1)STRUCTURAL ORGANIZATION IN ANIMALS (PART 1)
STRUCTURAL ORGANIZATION IN ANIMALS (PART 1)Shivankan Kakkar
 
Chemical Coordination and Integration NCERT
Chemical Coordination and Integration NCERTChemical Coordination and Integration NCERT
Chemical Coordination and Integration NCERTShivankan Kakkar
 
Calculate and Interpret Pharmacokinetic Parameters of a Given Drug
Calculate and Interpret Pharmacokinetic Parameters of a Given DrugCalculate and Interpret Pharmacokinetic Parameters of a Given Drug
Calculate and Interpret Pharmacokinetic Parameters of a Given DrugShivankan Kakkar
 
Effect of Drugs on Dog's Blood Pressure using CAL (Computer aided learning)
Effect of Drugs on Dog's Blood Pressure using CAL (Computer aided learning)Effect of Drugs on Dog's Blood Pressure using CAL (Computer aided learning)
Effect of Drugs on Dog's Blood Pressure using CAL (Computer aided learning)Shivankan Kakkar
 
Dosage Forms and Drug Delivery Systems
Dosage Forms and Drug Delivery SystemsDosage Forms and Drug Delivery Systems
Dosage Forms and Drug Delivery SystemsShivankan Kakkar
 
Essential Medicines List: What You Need to Know
Essential Medicines List: What You Need to KnowEssential Medicines List: What You Need to Know
Essential Medicines List: What You Need to KnowShivankan Kakkar
 
Understanding pA2 and pD2' Values: Calculation and Significance in Pharmacology
Understanding pA2 and pD2' Values: Calculation and Significance in PharmacologyUnderstanding pA2 and pD2' Values: Calculation and Significance in Pharmacology
Understanding pA2 and pD2' Values: Calculation and Significance in PharmacologyShivankan Kakkar
 
Mastering the Art of Setting up an IV Drip: A Step-by-Step Guide
Mastering the Art of Setting up an IV Drip: A Step-by-Step GuideMastering the Art of Setting up an IV Drip: A Step-by-Step Guide
Mastering the Art of Setting up an IV Drip: A Step-by-Step GuideShivankan Kakkar
 
One Liner Pharmacology RECALLS for NEET PG 2022
One Liner Pharmacology RECALLS for NEET PG 2022One Liner Pharmacology RECALLS for NEET PG 2022
One Liner Pharmacology RECALLS for NEET PG 2022Shivankan Kakkar
 
Mastering the Art of Prescribing IV Fluids: A Comprehensive Guide
Mastering the Art of Prescribing IV Fluids: A Comprehensive GuideMastering the Art of Prescribing IV Fluids: A Comprehensive Guide
Mastering the Art of Prescribing IV Fluids: A Comprehensive GuideShivankan Kakkar
 
Alpha Blockers: Mechanisms and Clinical Applications
Alpha Blockers: Mechanisms and Clinical ApplicationsAlpha Blockers: Mechanisms and Clinical Applications
Alpha Blockers: Mechanisms and Clinical ApplicationsShivankan Kakkar
 
Anatomy Question Bank for the NExT / NEET-PG 2023 Exam
Anatomy Question Bank for the NExT / NEET-PG 2023 ExamAnatomy Question Bank for the NExT / NEET-PG 2023 Exam
Anatomy Question Bank for the NExT / NEET-PG 2023 ExamShivankan Kakkar
 
NEET PG Recall- PART I.pptx
NEET PG Recall- PART I.pptxNEET PG Recall- PART I.pptx
NEET PG Recall- PART I.pptxShivankan Kakkar
 
Introduction to pharmacology
Introduction to pharmacologyIntroduction to pharmacology
Introduction to pharmacologyShivankan Kakkar
 
Hepatobiliary pancreatic surgery Set I Review
Hepatobiliary pancreatic surgery Set I ReviewHepatobiliary pancreatic surgery Set I Review
Hepatobiliary pancreatic surgery Set I ReviewShivankan Kakkar
 

More from Shivankan Kakkar (20)

STRUCTURAL ORGANIZATION IN ANIMALS (PART 1)
STRUCTURAL ORGANIZATION IN ANIMALS (PART 1)STRUCTURAL ORGANIZATION IN ANIMALS (PART 1)
STRUCTURAL ORGANIZATION IN ANIMALS (PART 1)
 
Chemical Coordination and Integration NCERT
Chemical Coordination and Integration NCERTChemical Coordination and Integration NCERT
Chemical Coordination and Integration NCERT
 
Calculate and Interpret Pharmacokinetic Parameters of a Given Drug
Calculate and Interpret Pharmacokinetic Parameters of a Given DrugCalculate and Interpret Pharmacokinetic Parameters of a Given Drug
Calculate and Interpret Pharmacokinetic Parameters of a Given Drug
 
Effect of Drugs on Dog's Blood Pressure using CAL (Computer aided learning)
Effect of Drugs on Dog's Blood Pressure using CAL (Computer aided learning)Effect of Drugs on Dog's Blood Pressure using CAL (Computer aided learning)
Effect of Drugs on Dog's Blood Pressure using CAL (Computer aided learning)
 
Dosage Forms and Drug Delivery Systems
Dosage Forms and Drug Delivery SystemsDosage Forms and Drug Delivery Systems
Dosage Forms and Drug Delivery Systems
 
Antiparkinsonian drugs
Antiparkinsonian drugsAntiparkinsonian drugs
Antiparkinsonian drugs
 
Essential Medicines List: What You Need to Know
Essential Medicines List: What You Need to KnowEssential Medicines List: What You Need to Know
Essential Medicines List: What You Need to Know
 
Understanding pA2 and pD2' Values: Calculation and Significance in Pharmacology
Understanding pA2 and pD2' Values: Calculation and Significance in PharmacologyUnderstanding pA2 and pD2' Values: Calculation and Significance in Pharmacology
Understanding pA2 and pD2' Values: Calculation and Significance in Pharmacology
 
Mastering the Art of Setting up an IV Drip: A Step-by-Step Guide
Mastering the Art of Setting up an IV Drip: A Step-by-Step GuideMastering the Art of Setting up an IV Drip: A Step-by-Step Guide
Mastering the Art of Setting up an IV Drip: A Step-by-Step Guide
 
One Liner Pharmacology RECALLS for NEET PG 2022
One Liner Pharmacology RECALLS for NEET PG 2022One Liner Pharmacology RECALLS for NEET PG 2022
One Liner Pharmacology RECALLS for NEET PG 2022
 
Mastering the Art of Prescribing IV Fluids: A Comprehensive Guide
Mastering the Art of Prescribing IV Fluids: A Comprehensive GuideMastering the Art of Prescribing IV Fluids: A Comprehensive Guide
Mastering the Art of Prescribing IV Fluids: A Comprehensive Guide
 
Alpha Blockers: Mechanisms and Clinical Applications
Alpha Blockers: Mechanisms and Clinical ApplicationsAlpha Blockers: Mechanisms and Clinical Applications
Alpha Blockers: Mechanisms and Clinical Applications
 
CAL Lab Pharmacology
CAL Lab PharmacologyCAL Lab Pharmacology
CAL Lab Pharmacology
 
Anatomy Question Bank for the NExT / NEET-PG 2023 Exam
Anatomy Question Bank for the NExT / NEET-PG 2023 ExamAnatomy Question Bank for the NExT / NEET-PG 2023 Exam
Anatomy Question Bank for the NExT / NEET-PG 2023 Exam
 
NEET PG MAY 2022
NEET PG MAY 2022NEET PG MAY 2022
NEET PG MAY 2022
 
NEET PG Recall- PART I.pptx
NEET PG Recall- PART I.pptxNEET PG Recall- PART I.pptx
NEET PG Recall- PART I.pptx
 
Introduction to pharmacology
Introduction to pharmacologyIntroduction to pharmacology
Introduction to pharmacology
 
Hepatobiliary pancreatic surgery Set I Review
Hepatobiliary pancreatic surgery Set I ReviewHepatobiliary pancreatic surgery Set I Review
Hepatobiliary pancreatic surgery Set I Review
 
Neurology review
Neurology reviewNeurology review
Neurology review
 
Neurology mcq test
Neurology mcq testNeurology mcq test
Neurology mcq test
 

Recently uploaded

Software Engineering Methodologies (overview)
Software Engineering Methodologies (overview)Software Engineering Methodologies (overview)
Software Engineering Methodologies (overview)eniolaolutunde
 
Presentation by Andreas Schleicher Tackling the School Absenteeism Crisis 30 ...
Presentation by Andreas Schleicher Tackling the School Absenteeism Crisis 30 ...Presentation by Andreas Schleicher Tackling the School Absenteeism Crisis 30 ...
Presentation by Andreas Schleicher Tackling the School Absenteeism Crisis 30 ...EduSkills OECD
 
SOCIAL AND HISTORICAL CONTEXT - LFTVD.pptx
SOCIAL AND HISTORICAL CONTEXT - LFTVD.pptxSOCIAL AND HISTORICAL CONTEXT - LFTVD.pptx
SOCIAL AND HISTORICAL CONTEXT - LFTVD.pptxiammrhaywood
 
Kisan Call Centre - To harness potential of ICT in Agriculture by answer farm...
Kisan Call Centre - To harness potential of ICT in Agriculture by answer farm...Kisan Call Centre - To harness potential of ICT in Agriculture by answer farm...
Kisan Call Centre - To harness potential of ICT in Agriculture by answer farm...Krashi Coaching
 
18-04-UA_REPORT_MEDIALITERAСY_INDEX-DM_23-1-final-eng.pdf
18-04-UA_REPORT_MEDIALITERAСY_INDEX-DM_23-1-final-eng.pdf18-04-UA_REPORT_MEDIALITERAСY_INDEX-DM_23-1-final-eng.pdf
18-04-UA_REPORT_MEDIALITERAСY_INDEX-DM_23-1-final-eng.pdfssuser54595a
 
Q4-W6-Restating Informational Text Grade 3
Q4-W6-Restating Informational Text Grade 3Q4-W6-Restating Informational Text Grade 3
Q4-W6-Restating Informational Text Grade 3JemimahLaneBuaron
 
Separation of Lanthanides/ Lanthanides and Actinides
Separation of Lanthanides/ Lanthanides and ActinidesSeparation of Lanthanides/ Lanthanides and Actinides
Separation of Lanthanides/ Lanthanides and ActinidesFatimaKhan178732
 
Micromeritics - Fundamental and Derived Properties of Powders
Micromeritics - Fundamental and Derived Properties of PowdersMicromeritics - Fundamental and Derived Properties of Powders
Micromeritics - Fundamental and Derived Properties of PowdersChitralekhaTherkar
 
Hybridoma Technology ( Production , Purification , and Application )
Hybridoma Technology  ( Production , Purification , and Application  ) Hybridoma Technology  ( Production , Purification , and Application  )
Hybridoma Technology ( Production , Purification , and Application ) Sakshi Ghasle
 
_Math 4-Q4 Week 5.pptx Steps in Collecting Data
_Math 4-Q4 Week 5.pptx Steps in Collecting Data_Math 4-Q4 Week 5.pptx Steps in Collecting Data
_Math 4-Q4 Week 5.pptx Steps in Collecting DataJhengPantaleon
 
Crayon Activity Handout For the Crayon A
Crayon Activity Handout For the Crayon ACrayon Activity Handout For the Crayon A
Crayon Activity Handout For the Crayon AUnboundStockton
 
URLs and Routing in the Odoo 17 Website App
URLs and Routing in the Odoo 17 Website AppURLs and Routing in the Odoo 17 Website App
URLs and Routing in the Odoo 17 Website AppCeline George
 
Concept of Vouching. B.Com(Hons) /B.Compdf
Concept of Vouching. B.Com(Hons) /B.CompdfConcept of Vouching. B.Com(Hons) /B.Compdf
Concept of Vouching. B.Com(Hons) /B.CompdfUmakantAnnand
 
CARE OF CHILD IN INCUBATOR..........pptx
CARE OF CHILD IN INCUBATOR..........pptxCARE OF CHILD IN INCUBATOR..........pptx
CARE OF CHILD IN INCUBATOR..........pptxGaneshChakor2
 
Arihant handbook biology for class 11 .pdf
Arihant handbook biology for class 11 .pdfArihant handbook biology for class 11 .pdf
Arihant handbook biology for class 11 .pdfchloefrazer622
 
Grant Readiness 101 TechSoup and Remy Consulting
Grant Readiness 101 TechSoup and Remy ConsultingGrant Readiness 101 TechSoup and Remy Consulting
Grant Readiness 101 TechSoup and Remy ConsultingTechSoup
 
Incoming and Outgoing Shipments in 1 STEP Using Odoo 17
Incoming and Outgoing Shipments in 1 STEP Using Odoo 17Incoming and Outgoing Shipments in 1 STEP Using Odoo 17
Incoming and Outgoing Shipments in 1 STEP Using Odoo 17Celine George
 
Introduction to ArtificiaI Intelligence in Higher Education
Introduction to ArtificiaI Intelligence in Higher EducationIntroduction to ArtificiaI Intelligence in Higher Education
Introduction to ArtificiaI Intelligence in Higher Educationpboyjonauth
 
Contemporary philippine arts from the regions_PPT_Module_12 [Autosaved] (1).pptx
Contemporary philippine arts from the regions_PPT_Module_12 [Autosaved] (1).pptxContemporary philippine arts from the regions_PPT_Module_12 [Autosaved] (1).pptx
Contemporary philippine arts from the regions_PPT_Module_12 [Autosaved] (1).pptxRoyAbrique
 

Recently uploaded (20)

Software Engineering Methodologies (overview)
Software Engineering Methodologies (overview)Software Engineering Methodologies (overview)
Software Engineering Methodologies (overview)
 
Model Call Girl in Tilak Nagar Delhi reach out to us at 🔝9953056974🔝
Model Call Girl in Tilak Nagar Delhi reach out to us at 🔝9953056974🔝Model Call Girl in Tilak Nagar Delhi reach out to us at 🔝9953056974🔝
Model Call Girl in Tilak Nagar Delhi reach out to us at 🔝9953056974🔝
 
Presentation by Andreas Schleicher Tackling the School Absenteeism Crisis 30 ...
Presentation by Andreas Schleicher Tackling the School Absenteeism Crisis 30 ...Presentation by Andreas Schleicher Tackling the School Absenteeism Crisis 30 ...
Presentation by Andreas Schleicher Tackling the School Absenteeism Crisis 30 ...
 
SOCIAL AND HISTORICAL CONTEXT - LFTVD.pptx
SOCIAL AND HISTORICAL CONTEXT - LFTVD.pptxSOCIAL AND HISTORICAL CONTEXT - LFTVD.pptx
SOCIAL AND HISTORICAL CONTEXT - LFTVD.pptx
 
Kisan Call Centre - To harness potential of ICT in Agriculture by answer farm...
Kisan Call Centre - To harness potential of ICT in Agriculture by answer farm...Kisan Call Centre - To harness potential of ICT in Agriculture by answer farm...
Kisan Call Centre - To harness potential of ICT in Agriculture by answer farm...
 
18-04-UA_REPORT_MEDIALITERAСY_INDEX-DM_23-1-final-eng.pdf
18-04-UA_REPORT_MEDIALITERAСY_INDEX-DM_23-1-final-eng.pdf18-04-UA_REPORT_MEDIALITERAСY_INDEX-DM_23-1-final-eng.pdf
18-04-UA_REPORT_MEDIALITERAСY_INDEX-DM_23-1-final-eng.pdf
 
Q4-W6-Restating Informational Text Grade 3
Q4-W6-Restating Informational Text Grade 3Q4-W6-Restating Informational Text Grade 3
Q4-W6-Restating Informational Text Grade 3
 
Separation of Lanthanides/ Lanthanides and Actinides
Separation of Lanthanides/ Lanthanides and ActinidesSeparation of Lanthanides/ Lanthanides and Actinides
Separation of Lanthanides/ Lanthanides and Actinides
 
Micromeritics - Fundamental and Derived Properties of Powders
Micromeritics - Fundamental and Derived Properties of PowdersMicromeritics - Fundamental and Derived Properties of Powders
Micromeritics - Fundamental and Derived Properties of Powders
 
Hybridoma Technology ( Production , Purification , and Application )
Hybridoma Technology  ( Production , Purification , and Application  ) Hybridoma Technology  ( Production , Purification , and Application  )
Hybridoma Technology ( Production , Purification , and Application )
 
_Math 4-Q4 Week 5.pptx Steps in Collecting Data
_Math 4-Q4 Week 5.pptx Steps in Collecting Data_Math 4-Q4 Week 5.pptx Steps in Collecting Data
_Math 4-Q4 Week 5.pptx Steps in Collecting Data
 
Crayon Activity Handout For the Crayon A
Crayon Activity Handout For the Crayon ACrayon Activity Handout For the Crayon A
Crayon Activity Handout For the Crayon A
 
URLs and Routing in the Odoo 17 Website App
URLs and Routing in the Odoo 17 Website AppURLs and Routing in the Odoo 17 Website App
URLs and Routing in the Odoo 17 Website App
 
Concept of Vouching. B.Com(Hons) /B.Compdf
Concept of Vouching. B.Com(Hons) /B.CompdfConcept of Vouching. B.Com(Hons) /B.Compdf
Concept of Vouching. B.Com(Hons) /B.Compdf
 
CARE OF CHILD IN INCUBATOR..........pptx
CARE OF CHILD IN INCUBATOR..........pptxCARE OF CHILD IN INCUBATOR..........pptx
CARE OF CHILD IN INCUBATOR..........pptx
 
Arihant handbook biology for class 11 .pdf
Arihant handbook biology for class 11 .pdfArihant handbook biology for class 11 .pdf
Arihant handbook biology for class 11 .pdf
 
Grant Readiness 101 TechSoup and Remy Consulting
Grant Readiness 101 TechSoup and Remy ConsultingGrant Readiness 101 TechSoup and Remy Consulting
Grant Readiness 101 TechSoup and Remy Consulting
 
Incoming and Outgoing Shipments in 1 STEP Using Odoo 17
Incoming and Outgoing Shipments in 1 STEP Using Odoo 17Incoming and Outgoing Shipments in 1 STEP Using Odoo 17
Incoming and Outgoing Shipments in 1 STEP Using Odoo 17
 
Introduction to ArtificiaI Intelligence in Higher Education
Introduction to ArtificiaI Intelligence in Higher EducationIntroduction to ArtificiaI Intelligence in Higher Education
Introduction to ArtificiaI Intelligence in Higher Education
 
Contemporary philippine arts from the regions_PPT_Module_12 [Autosaved] (1).pptx
Contemporary philippine arts from the regions_PPT_Module_12 [Autosaved] (1).pptxContemporary philippine arts from the regions_PPT_Module_12 [Autosaved] (1).pptx
Contemporary philippine arts from the regions_PPT_Module_12 [Autosaved] (1).pptx
 

Zoology Question Bank for the NEET-UG 2024 Exam

  • 1. <a href="https://www.freepik.com/free-photo/eco-message-bottle- concept_31760605.htm#fromView=search&page=1&position=3&uuid=faa0320f-8519-42e1-a493-88cbb342ec46">Image by freepik</a> ZOOLOGY QUESTION BANK FOR THE NEET-UG 2024 EXAM Dr. Shivankan Kakkar, MBBS, MD (Gold Medal) FLASHCARDS This is a set of 50 flashcards that contains zoology questions from the NEET-UG 2023 exam. LINKED LEARNING You can use these flashcards to test your knowledge and prepare for the exam.
  • 2. Dr. Shivankan Kakkar, MBBS, MD (Gold Medal) SECTION A 1. Which of the following statements are correct regarding female reproductive cycle? A. In non-primate mammals cyclical changes during reproduction are called oestrus cycle. B. First menstrual cycle begins at puberty and is called menopause. C. Lack of menstruation may be indicative of pregnancy. D. Cyclic menstruation extends between menarche and menopause. Choose the most appropriate answer from the options given below. (1) A and B only (2) A, B and Conly (3) A, C and D only (4) A and D only Answer (3)
  • 3. Dr. Shivankan Kakkar, MBBS, MD (Gold Medal) • Statement A: This statement is true. In non-primate mammals, cyclical changes during reproduction are called oestrus cycle. This is a type of reproductive cycle in which females are sexually receptive only during a specific period called oestrus or heat1 . During oestrus, the female exhibits behavioural and physiological changes that attract males for mating1 . • Statement B: This statement is false. The first menstrual cycle begins at puberty and is called menarche, not menopause. Menarche is the onset of menstruation in females, which usually occurs between the ages of 11 and 152 . Menopause is the cessation of menstruation in females, which usually occurs between the ages of 45 and 552 . • Statement C: This statement is true. Lack of menstruation may be indicative of pregnancy, as one of the earliest signs of pregnancy is a missed period3 . However, there may be other causes of amenorrhea (absence of menstruation), such as hormonal imbalance, stress, illness, or medication3 . • Statement D: This statement is true. Cyclic menstruation extends between menarche and menopause, which are the two milestones that mark the beginning and the end of the reproductive years in females2 . The menstrual cycle is a monthly cycle of hormonal and physiological changes that prepare the female body for pregnancy2 . Therefore, the correct answer is (3) A, C and D only.
  • 4. Dr. Shivankan Kakkar, MBBS, MD (Gold Medal) 2. Given below are two statements: one is labelled as Assertion A and other is labelled as Reason R. Assertion A : Amniocentesis for sex determination is one of the strategies of Reproductive and Child Health Care Programme. Reason R : Ban on amniocentesis checks increasing menace of female foeticide. In the light of the above statements, choose the correct answer from the options given below. (1) Both A and R are true and R is NOT the correct explanation of A. (2) A is true but R is false. (3) A is false but R is true. (4) Both A and R are true and R is the correct explanation of A. Answer (3)
  • 5. Dr. Shivankan Kakkar, MBBS, MD (Gold Medal) • Assertion A: This statement is false. Amniocentesis is not a strategy of Reproductive and Child Health Care Programme, but a prenatal diagnostic technique that can detect the sex and genetic abnormalities of the fetus1 . However, this technique has been misused for sex determination and female foeticide, which is the deliberate abortion of a female fetus23 . Therefore, amniocentesis is not a part of the reproductive and child health care programme, but a tool that can be abused for gender discrimination. • Reason R: This statement is true. Ban on amniocentesis checks the increasing menace of female foeticide, as it prevents the illegal use of this technique for sex selection and abortion. The Indian government has passed the Pre- Conception and Pre-Natal Diagnostic Techniques (PCPNDT) Act in 1994 to prohibit and punish the misuse of amniocentesis and other prenatal diagnostic methods for sex determination and female foeticide4 . The ban on amniocentesis is aimed at protecting the rights and dignity of the girl child and ensuring gender equality. Therefore, the correct answer is (3) A is false but R is true.
  • 6. Dr. Shivankan Kakkar, MBBS, MD (Gold Medal) 3. Match List I with List II. List I List II (Interacting species) (Name of interaction) A. A Leopard and a Lion in a forest/grassland I. Competition B. A Cuckoo laying egg in a Crow’s nest II. Brood parasitism C. Fungi and root of a higher plant in Mycorrhizae III. Mutualism D. A cattle egret and a Cattle in a field IV. Commensalism Choose the correct answer from the options given below. (1) A-I, B-II, C-IV, D-III (2) A-III, B-IV, C-I, D-II (3) A-II, B-III, C-I, D-IV (4) A-I, B-II, C-III, D-IV Answer (4)
  • 7. Dr. Shivankan Kakkar, MBBS, MD (Gold Medal) The matching of the interacting species with the name of interaction is as follows: • A. A Leopard and a Lion in a forest/grassland: This is an example of competition (I), which is a type of interaction where two or more species use the same limited resource, such as food, water, space, or mates, and negatively affect each other’s fitness12 . Leopards and lions are both carnivorous predators that hunt similar prey and may compete for food in the same habitat3 . • B. A Cuckoo laying egg in a Crow’s nest: This is an example of brood parasitism (II), which is a type of interaction where one species lays its eggs in the nest of another species and relies on the host to raise its offspring, often at the expense of the host’s own offspring12 . Cuckoos are notorious brood parasites that exploit various species of birds, such as crows, by laying eggs that mimic the host’s eggs and removing or destroying some of the host’s eggs4 . • C. Fungi and root of a higher plant in Mycorrhizae: This is an example of mutualism (III), which is a type of interaction where two or more species benefit from each other and enhance each other’s fitness12 . Mycorrhizae are symbiotic associations between fungi and the roots of higher plants, where the fungi help the plants absorb water and minerals from the soil, and the plants provide the fungi with organic nutrients. • D. A cattle egret and a Cattle in a field: This is an example of commensalism (IV), which is a type of interaction where one species benefits from another species, but the other species is neither helped nor harmed12 . Cattle egrets are birds that follow cattle and other large herbivores in fields, and feed on the
  • 8. Dr. Shivankan Kakkar, MBBS, MD (Gold Medal) insects that are disturbed by the cattle’s grazing. The cattle egrets gain access to food, but the cattle are unaffected by their presence. Therefore, the correct answer is (4) A-I, B-II, C-III, D-IV.
  • 9. Dr. Shivankan Kakkar, MBBS, MD (Gold Medal) 4. Vital capacity of lung is _________. (1) IRV + ERV + TV + RV (2) IRV + ERV + TV –RV (3) IRV + ERV + TV (4) IRV + ERV Answer (3)
  • 10. Dr. Shivankan Kakkar, MBBS, MD (Gold Medal) Vital capacity of lung is the total amount of air that can be exhaled after a maximum inhalation. It is equal to the sum of inspiratory reserve volume (IRV), tidal volume (TV), and expiratory reserve volume (ERV). It is an important measure of a person’s respiratory health. A decreased vital capacity is an indication of restrictive lung disease, where the lungs cannot expand fully. One of the links that explains this is here. This is a web page that talks about the lung volumes and capacities, such as tidal volume, residual volume, vital capacity, inspiratory capacity, etc.
  • 11. Dr. Shivankan Kakkar, MBBS, MD (Gold Medal) 5. Which one of the following common sexually transmitted diseases is completely curable when detected early and treated properly? (1) Gonorrhoea (2) Hepatitis-B (3) HIV Infection (4) Genital herpes Answer (1)
  • 12. Dr. Shivankan Kakkar, MBBS, MD (Gold Medal) Gonorrhoea is a bacterial infection that can be cured with antibiotics. Hepatitis-B, HIV infection, and genital herpes are viral infections that are not completely curable. One of the links that explains this is here.
  • 13. Dr. Shivankan Kakkar, MBBS, MD (Gold Medal) 6. Match List I with List II. List I List II A. CCK I. Kidney B. GIP II. Heart C. ANF III. Gastric gland D. ADH IV. Pancreas Choose the correct answer from the options given below : (1) A-III, B-II, C-IV, D-I (2) A-II, B-IV, C-I, D-III (3) A-IV, B-II, C-III, D-I (4) A-IV, B-III, C-II, D-I Answer (4)
  • 14. Dr. Shivankan Kakkar, MBBS, MD (Gold Medal) The correct answer is (4) A-IV, B-III, C-II, D-I. Here is a brief explanation of the matching of the hormones with their sources: • CCK (A): CCK stands for cholecystokinin, a hormone that is secreted by the cells of the duodenum and jejunum in response to the presence of fats and proteins in the small intestine. It stimulates the gallbladder to release bile and the pancreas to secrete digestive enzymes. It matches with Pancreas (IV)12 . • GIP (B): GIP stands for gastric inhibitory peptide, a hormone that is secreted by the cells of the duodenum in response to the presence of glucose and fats in the small intestine. It inhibits the secretion of gastric acid and motility, and stimulates the secretion of insulin. It matches with Gastric gland (III)13 . • ANF ©: ANF stands for atrial natriuretic factor, a hormone that is secreted by the cells of the atria of the heart in response to high blood pressure or volume. It causes the kidneys to excrete more sodium and water, and lowers the blood pressure. It matches with Heart (II)4 . • ADH (D): ADH stands for antidiuretic hormone, a hormone that is secreted by the posterior pituitary gland in response to low blood volume or high blood osmolarity. It causes the kidneys to reabsorb more water, and increases the blood volume and pressure. It matches with Kidney (I)4 .
  • 15. Dr. Shivankan Kakkar, MBBS, MD (Gold Medal) 7. Match List I with List II. List I List II A. Ringworm I. Haemophilus influenzae B. Filariasis II. Trichophyton C. Malaria III. Wuchereria bancrofti D. Pneumonia IV. Plasmodium vivax Choose the correct answer from the options given below : (1) A-II, B-III, C-I, D-IV (2) A-III, B-II, C-I, D-IV (3) A-III, B-II, C-IV, D-I (4) A-II, B-III, C-IV, D-I Answer (4)
  • 16. Dr. Shivankan Kakkar, MBBS, MD (Gold Medal) The correct answer is (4) A-II, B-III, C-IV, D-I. Here is a brief explanation of the matching of the diseases with their causes: • Ringworm (A): Ringworm is a common skin infection that is caused by a fungus. It’s usually an itchy, circular rash with clearer skin in the middle. It matches with Trichophyton (II), which is one of the three types of fungi that cause ringworm12345 . • Filariasis (B): Filariasis is a parasitic disease that is caused by infection with thread-like worms called filariae. It can cause swelling of the limbs, genitals, and breasts, and can lead to elephantiasis. It matches with Wuchereria bancrofti (III), which is the most common cause of lymphatic filariasis. • Malaria ©: Malaria is a life-threatening disease that is caused by infection with a parasite called Plasmodium. It is transmitted by the bite of an infected mosquito. It can cause fever, chills, headache, nausea, vomiting, and other symptoms. It matches with Plasmodium vivax (IV), which is one of the five species of Plasmodium that cause malaria in humans. • Pneumonia (D): Pneumonia is an infection that inflames the air sacs in one or both lungs. It can be caused by bacteria, viruses, fungi, or other organisms. It can cause cough, fever, difficulty breathing, and chest pain. It matches with Haemophilus influenzae (I), which is one of the common bacterial causes of pneumonia.
  • 17. Dr. Shivankan Kakkar, MBBS, MD (Gold Medal) 8. Match List I with List II. List I List II A. P-wave I. Beginning of systole B. Q-wave II. Repolarisation of ventricles C. QRS complex III. Depolarisation of atria D. T-wave IV. Depolarisation of ventricles Choose the correct answer from the options given below : (1) A-IV, B-III, C-II, D-I (2) A-II, B-IV, C-I, D-III (3) A-I, B-II, C-III, D-IV (4) A-III, B-I, C-IV, D-II Answer (4)
  • 18. Dr. Shivankan Kakkar, MBBS, MD (Gold Medal) The correct answer is (4) A-III, B-I, C-IV, D-II. Here is a brief explanation of the matching of the ECG waves with their corresponding events: • P-wave (A): The P-wave reflects atrial depolarization, which is the electrical activation of the atria. It occurs before the atria contract and push blood into the ventricles. It matches with Depolarisation of atria (III)1 . • Q-wave (B): The Q-wave is the first negative deflection of the QRS complex. It represents the depolarization of the interventricular septum, which is the muscular wall that separates the left and right ventricles. It matches with Beginning of systole (I)2 , which is the phase of the cardiac cycle when the ventricles contract and eject blood. • QRS complex ©: The QRS complex is the most prominent feature of the ECG. It consists of three waves: Q, R, and S. It represents the depolarization of the ventricles, which is the electrical activation of the ventricular myocardium. It matches with Depolarisation of ventricles (IV)2 . • T-wave (D): The T-wave is the positive deflection that follows the QRS complex. It represents the repolarization of the ventricles, which is the restoration of the electrical potential of the ventricular myocardium after contraction. It matches with Repolarisation of ventricles (II)2 .
  • 19. Dr. Shivankan Kakkar, MBBS, MD (Gold Medal) 9. In which blood corpuscles, the HIV undergoes replication and produces progeny viruses? (1) B-lymphocytes (2) Basophils (3) Eosinophils (4) T helper cells Answer (4)
  • 20. Dr. Shivankan Kakkar, MBBS, MD (Gold Medal) HIV undergoes replication and produces progeny viruses in T helper cells, which are a type of white blood cell that helps other immune cells to fight infections. • HIV - Biology: AQA A Level
  • 21. Dr. Shivankan Kakkar, MBBS, MD (Gold Medal) 10. Given below are two statements : Statement I : Low temperature preserves the enzyme in a temporarily inactive state whereas high temperature destroys enzymatic activity because proteins are denatured by heat. Statement II : When the inhibitor closely resembles the substrate in its molecular structure and inhibits the activity of the enzyme, it is known as competitive inhibitor. In the light of the above statements, choose the correct answer from the options given below : (1) Both Statement I and Statement II are false. (2) Statement I is true but Statement II is flase. (3) Statement I is false but Statement II is true. (4) Both Statement I and Statement II are true. Answer (4)
  • 22. Dr. Shivankan Kakkar, MBBS, MD (Gold Medal) • Statement I: This statement is true. Low temperature preserves the enzyme in a temporarily inactive state because it reduces the kinetic energy of the molecules and slows down the rate of reaction. However, low temperature does not affect the structure or function of the enzyme, and the enzyme can resume its activity when the temperature is increased1 . High temperature destroys enzymatic activity because proteins are denatured by heat. This means that the high temperature disrupts the bonds that maintain the three-dimensional shape of the enzyme, and thus alters its active site and prevents it from binding to the substrate1 . • Statement II: This statement is true. When the inhibitor closely resembles the substrate in its molecular structure and inhibits the activity of the enzyme, it is known as a competitive inhibitor. A competitive inhibitor competes with the substrate for the same active site of the enzyme, and thus reduces the number of enzyme-substrate complexes formed2 . The effect of competitive inhibition can be overcome by increasing the concentration of the substrate, which increases the chances of the substrate binding to the enzyme2 . Therefore, the correct answer is (4) Both Statement I and Statement II are true.
  • 23. Dr. Shivankan Kakkar, MBBS, MD (Gold Medal) 11. Given below are two statements: Statement I: Ligaments are dense irregular tissue. Statement II: Cartilage is dense regular tissue. In the light of the above statements, choose the correct answer from the options given below: (1) Both Statement I and Statement II are false (2) Statement I is true but Statement II is false (3) Statement I is false but Statement II is true (4) Both Statement I and Statement II are true Answer (1)
  • 24. Dr. Shivankan Kakkar, MBBS, MD (Gold Medal) The correct answer is (1) Both Statement I and Statement II are false. Ligaments are not dense irregular tissue, but dense regular tissue. They are composed of parallel bundles of collagen fibers that connect bones to bones and provide strength and stability to joints. Cartilage is not dense regular tissue, but a type of specialized connective tissue. It is composed of a dense network of collagen and elastic fibers embedded in a firm, gelatinous ground substance. It is found at the end of joints, the rib cage, the ear, the nose, in the throat, and between intervertebral disks. • Dense connective tissue: Cells, fibers and types • Dense irregular connective tissue: anatomy and function • Cartilage | Description, Anatomy, & Function
  • 25. Dr. Shivankan Kakkar, MBBS, MD (Gold Medal) 12. Which of the following are NOT considered as the part of endomembrane system? A. Mitochondria B. Endoplasmic reticulum C. Chloroplasts D. Golgi complex E. Peroxisomes Choose the most appropriate answer from the options given below: (1) A,C and E only (2) A and D only (3) A,D and E only (4) B and D only Answer (1)
  • 26. Dr. Shivankan Kakkar, MBBS, MD (Gold Medal) The correct answer is indeed (1) A,C and E only. The endomembrane system is a group of membranes and organelles in eukaryotic cells that work together to modify, package, and transport lipids and proteins. It includes the nuclear envelope, the endoplasmic reticulum, the Golgi complex, lysosomes, endosomes, vesicles, and the plasma membrane1 . Mitochondria, chloroplasts, and peroxisomes are not part of the endomembrane system because they have their own membranes and are not derived from the endoplasmic reticulum2 .
  • 27. Dr. Shivankan Kakkar, MBBS, MD (Gold Medal) 13. Given below are two statements: Statement I: A protein is imagined as a line, the left end represented by first amino acid (C-terminal) and the right end represented by last amino acid (N-terminal). Statement II: Adult human haemoglobin, consists of 4 subunits (two subunits of a type and two subunits of ß type.) In the light of the above statements, choose the correct answer from the options given below: (1) Both Statement I and Statement II are false. (2) Statement I is true but Statement II is false. (3) Statement I is false but Statement II is true. (4) Both Statement I and Statement II are true. Answer (3)
  • 28. Dr. Shivankan Kakkar, MBBS, MD (Gold Medal) • Statement I: This statement is false. A protein is imagined as a line, the left end represented by the first amino acid (N-terminal) and the right end represented by the last amino acid (C-terminal)12 . The N-terminal is the end of the protein chain that has a free amino group, while the C-terminal is the end that has a free carboxyl group12 . • Statement II: This statement is true. Adult human haemoglobin consists of four subunits (two subunits of α type and two subunits of β type)34 . Each subunit contains a heme group, which is a complex of an iron atom and a porphyrin ring, that can bind oxygen reversibly34 . Therefore, the correct answer is (3), as statement I is false but statement II is true.
  • 29. Dr. Shivankan Kakkar, MBBS, MD (Gold Medal) 14. Broad palm with single palm crease is visible in a person suffering from­ (1) Turner’s syndrome (2) Klinefelter’s syndrome (3) Thalassemia (4) Down’s syndrome Answer (4)
  • 30. Dr. Shivankan Kakkar, MBBS, MD (Gold Medal) Some key points on the single transverse palmar crease, also known as the simian crease: • A single transverse palmar crease is a single crease that extends across the palm of the hand, formed by the fusion of the two palmar creases1 . • It is found at a higher frequency in people with certain disorders, such as Down syndrome, fetal alcohol syndrome and others1 . • Down syndrome is a genetic disorder caused by the presence of an extra copy of chromosome 21. It causes intellectual disabilities, a characteristic facial appearance, and an increased chance for heart defects and digestive issues2 . • A broad palm with a single palm crease is a characteristic physical feature commonly associated with individuals who have Down syndrome3 . However, not all people with Down syndrome have this feature, and thus it is not a diagnostic indicator of the disorder1 . Therefore, the correct answer is (4), as broad palm with single palm crease is visible in a person suffering from Down syndrome.
  • 31. Dr. Shivankan Kakkar, MBBS, MD (Gold Medal) 15. Which of the follwoing statements is correct? (1) Biomagnification refers to increase in concentration of the toxicant at successive trophic levels. (2) Presence of large amount of nutrients in water restricts ‘Algal Bloom’ (3) Algal Bloom decreases fish mortality (4) Eutrophication refers to increase in domestic sewage and waste water in lakes. Answer (1)
  • 32. Dr. Shivankan Kakkar, MBBS, MD (Gold Medal) • Biomagnification is the process of accumulation of certain chemicals in living organisms to a concentration higher than that occurring in the inorganic, non- living environment. It can be caused by agriculture, organic contaminants, industrial activities and mining activities. It has negative impacts on human health, reproduction, coral reefs and food chain. • Algal bloom is a rapid increase or accumulation in the population of algae in freshwater or marine water systems. It is often recognized by the discoloration in the water from the algae’s pigments. It can be caused by human activities such as fertilizers, sewage, or industrial wastes, or by natural events such as floods or erosion. Algal bloom has serious environmental and ecological consequences, such as reducing biodiversity, depleting oxygen, and creating dead zones. • Eutrophication is the process of excessively enriching a water body in nutrients, primarily phosphorus and nitrogen, leading to the plentiful growth of simple plant life. This process can significantly disrupt aquatic ecosystems and is influenced by a variety of factors, including agricultural runoff, industrial discharge, and urbanization. Eutrophication can result in algal blooms, fish kills, and loss of aquatic habitats. Therefore, the correct answer is (1), as biomagnification refers to the increase in concentration of the toxicant at successive trophic levels. The other statements are incorrect, as presence of large amount of nutrients in water promotes algal bloom, algal bloom increases fish mortality, and eutrophication refers to the enrichment of water body in nutrients, not sewage and waste water.
  • 33. Dr. Shivankan Kakkar, MBBS, MD (Gold Medal) 16. Match List I with List II. List I List II A. Taenia I. Nephridia B. Paramoecium II. Contractile vacuole C. Periplaneta III. Flame cells D. Pheretima IV. Urecose gland Choose the correct answer from the options given below: (1) A-I, B-II, C-IV, D-III (2) A-III, B-II, C-IV, D-I (3) A-II, B-I, C-IV, D-III (4) A-I, B-II, C-III, D-IV Answer (2)
  • 34. Dr. Shivankan Kakkar, MBBS, MD (Gold Medal) • Taenia (A): Taenia is a genus of tapeworms. The excretory organs in Taenia are flame cells (III) which are scattered all over the mesenchyme12345 . • Paramoecium (B): Paramoecium is a genus of unicellular ciliates. It has a contractile vacuole (II) that helps regulate the water content within the cell6789 . • Periplaneta ©: Periplaneta is a genus of cockroaches. It has urecose glands (IV) as part of its excretory system101112 . • Pheretima (D): Pheretima is a genus of earthworms. It has nephridia (I) as part of its excretory system131415161718 . So, the correct answer is (2) A-III, B-II, C-IV, D-I.
  • 35. Dr. Shivankan Kakkar, MBBS, MD (Gold Medal) 17. Given below are two statements: Statement I:Electrostatic precipitator is most widely used in thermal power plant . Statement II: Electrostatic precipitator in thermal power plant removes ionising radiations. In the light of the above statements, choose the most appropriate answer from the options given below: (1) Both Statement I and Statement II are incorrect. (2) Statement I is correct but Statement II is incorrect. (3) Statement I is incorrect but Statement II is correct. (4) Both Statement I and Statement II are correct. Answer (2)
  • 36. Dr. Shivankan Kakkar, MBBS, MD (Gold Medal) • Statement I: An electrostatic precipitator (ESP) is a filtration device that is used to remove fine particles like smoke and fine dust from the flowing gas12345 . It is the most commonly used device for air pollution control and is widely used in thermal power plants12345 . • Statement II: However, electrostatic precipitators do not remove ionizing radiations67 . They work by using electrostatic charges to attract and trap particles on charged plates or tubes67 . So, the correct answer is (2) Statement I is correct but Statement II is incorrect.
  • 37. Dr. Shivankan Kakkar, MBBS, MD (Gold Medal) 18. Given below are two statements: Statement I: Vas deferens receives a duct from seminal vesicle and opens into urethra as the ejaculatory duct. Statement Il: The cavity of the cervix is called cervical canal which along with vagina forms birth canal. In the light of the above statements, choose the correct answer from the options given below: (1) Both Statement I and Statement II are false. (2) Statement I is correct but Statement II is false. (3) Statement I is incorrect but Statement II is true. (4) Both Statement I and Statement II are true. Answer (4)
  • 38. Dr. Shivankan Kakkar, MBBS, MD (Gold Medal) • Statement I: The vas deferens receives a duct from the seminal vesicle and opens into the urethra as the ejaculatory duct12345 . The vas deferens is a continuation of the cauda epididymis (tail part of epididymis). It is about 40 cm long and slightly coiled at first but becomes straight as it enters the abdominal cavity through the inguinal canal. Here, it passes over the urinary bladder, curves around the ureter and joins a duct from the seminal vesicle and opens into the urethra as the ejaculatory duct12345 . • Statement II: The cavity of the cervix is called the cervical canal which along with the vagina forms the birth canal678910 . The cervical canal communicates with the uterine cavity via the internal orifice of the uterus (or internal os) and with the vagina via the external orifice of the uterus (ostium of uterus or external os). The cervical canal, along with the vagina, forms the birth canal through which a baby passes during childbirth678910 . So, the correct answer is (4) Both Statement I and Statement II are true.
  • 39. Dr. Shivankan Kakkar, MBBS, MD (Gold Medal) 19. Radial symmetry is NOT found in adults of phylum ______. (1) Hemichordata (2) Coelenterata (3) Echinodermata (4) Ctenophora Answer (1)
  • 40. Dr. Shivankan Kakkar, MBBS, MD (Gold Medal) The correct answer is (1) Hemichordata. Radial symmetry is NOT found in adults of phylum Hemichordata, a group of marine deuterostomes that are bilaterally symmetrical.
  • 41. Dr. Shivankan Kakkar, MBBS, MD (Gold Medal) 20. Match List I with List II . List I List II (Cells) (Secretion) A. Peptic cells I. Mucus B. Goblet cells II. Bile juice C. Oxyntic cells III. Proenzyme pepsinogen D. Hepatic cells IV. HCl & intrinsic factor for absorption of vit B12 Choose the correct answer from the options given below: (1) A-II, B-I, C-III, D-IV (2) A-III, B-I, C-IV, D-II (3) A-II, B-IV, C-I, D-III (4) A-IV, B-III, C-II, D-I Answer (2)
  • 42. Dr. Shivankan Kakkar, MBBS, MD (Gold Medal) • Peptic cells (A): These cells secrete the proenzyme pepsinogen, which is later activated to the enzyme pepsin and helps in the digestion of proteins12 . So, they match with Proenzyme pepsinogen (III)12 . • Goblet cells (B): These are a specialized type of epithelial cell that secrete mucins, which are significant components of mucus34 . They are most often found in the respiratory and gastrointestinal tracts, where they make up part of the surface epithelium34 . So, they match with Mucus (I)34 . • Oxyntic cells ©: Also known as parietal cells, these cells secrete hydrochloric acid and intrinsic factor, which is necessary for the absorption of vitamin B1212 . So, they match with HCl & intrinsic factor for absorption of vit B12 (IV)12 . • Hepatic cells (D): These are the main functional cells of the liver and are involved in protein synthesis, protein storage and transformation of carbohydrates, synthesis of cholesterol, bile salts and phospholipids, and detoxification, modification and excretion of exogenous and endogenous substances. They secrete bile juice1 . So, they match with Bile juice (II)1 . So, the correct answer is (2) A-III, B-I, C-IV, D-II.
  • 43. Dr. Shivankan Kakkar, MBBS, MD (Gold Medal) 21. Match List I with List II with respect in human eye. List I List II A. Fovea I. Visible coloured portion of eye that regulates diameter of pupil. B. Iris II. External layer of eye formed of dense connective tissue. C. Blind spot III. Point of greatest visual acuity or resolution. D. Sclera IV. Point where optic nerve leaves the eyeball and photoreceptor cells are absent. Choose the correct answer from the options given below: (1) A-IV, B-III, C-II, D-I (2) A-I, B-IV, C-III, D-II (3) A-II, B-I, C-III, D-IV (4) A-III, B-I, C-IV, D-II Answer (4)
  • 44. Dr. Shivankan Kakkar, MBBS, MD (Gold Medal) • Fovea (A): The fovea is a small depression in the retina where visual acuity is highest. It is the point of greatest visual acuity or resolution (III)12 . • Iris (B): The iris is the colored part of the eye that controls the amount of light that enters the eye. It is the visible colored portion of the eye that regulates the diameter of the pupil (I)12 . • Blind spot ©: The blind spot is the point in the visual field that corresponds to the lack of light-detecting photoreceptor cells on the optic disc of the retina where the optic nerve passes through the optic disc. It is the point where the optic nerve leaves the eyeball and photoreceptor cells are absent (IV)12 . • Sclera (D): The sclera is the white outer layer of the eye. It is the external layer of the eye formed of dense connective tissue (II)12 . So, the correct answer is (4) A-III, B-I, C-IV, D-II.
  • 45. Dr. Shivankan Kakkar, MBBS, MD (Gold Medal) 22. Which of the following functions is carried out by cytoskeleton in a cell? (1) Protein synthesis (2) Motility (3) Transportation (4) Nuclear division Answer (2)
  • 46. Dr. Shivankan Kakkar, MBBS, MD (Gold Medal) The cytoskeleton in a cell carries out several functions12 . 1.Protein Synthesis: The cytoskeleton itself does not directly participate in protein synthesis. Protein synthesis is primarily carried out by ribosomes, which are not part of the cytoskeleton. 2.Motility: Yes, the cytoskeleton plays a crucial role in cell motility12 . Microfilaments, a component of the cytoskeleton, aid in cell motility and allow single-celled organisms like amoebas to move1 . 3.Transportation: The cytoskeleton facilitates the transport of molecules within the cell12 . Microfilaments are involved in cytoplasmic streaming, which is the flowing of cytosol (the liquid part of the cytoplasm) throughout the cell, transporting nutrients and cell organelles1 . 4.Nuclear Division: While the cytoskeleton does play a role in cell division, it does not directly carry out nuclear division. However, microtubules, another component of the cytoskeleton, help in dividing chromosomes during cell division2 .
  • 47. Dr. Shivankan Kakkar, MBBS, MD (Gold Medal) 23. Once the undigested and unabsorbed substances enter the caecum, their backflow is prevented by (1) Ileo-caecal valve (2) Gastro-oesophageal sphincter (3) Pyloric sphincter (4) Sphincter of Oddi Answer (1)
  • 48. Dr. Shivankan Kakkar, MBBS, MD (Gold Medal) The Ileo-caecal valve (1) is a sphincter muscle valve that separates the small intestine and the large intestine. Its critical function is to limit the reflux of colonic contents into the ileum1 . This valve is located at the junction between the ileum and the cecum1 . So, the correct answer is (1) Ileo-caecal valve.
  • 49. Dr. Shivankan Kakkar, MBBS, MD (Gold Medal) 24. Match List I with List II. List I List II A. Vasectomy I. Oral method B. Coitus interruptus II. Barrier method C. Cervical caps III. Surgical method D. Saheli IV. Natural method Choose the correct answer from the options given below: (1) A-III, B-IV, C-II, D-I (2) A-II, B-III, C-I, D-IV (3) A-IV, B-II, C-I, D-III (4) A-III, B-I, C-IV, D-II Answer (1)
  • 50. Dr. Shivankan Kakkar, MBBS, MD (Gold Medal) • Vasectomy (A): This is a surgical procedure that makes a person who can produce sperm unable to cause a pregnancy1 . It is a surgical method (III)1 . • Coitus interruptus (B): Also known as the withdrawal or “pull out” method, it involves withdrawing the penis from the vagina before ejaculation. It is considered a natural method (IV)1 . • Cervical caps ©: These are small, thimble-shaped cups made of silicone. They are inserted into the vagina before sex to prevent pregnancy. They are considered a barrier method (II)1 . • Saheli (D): This is a non-hormonal oral contraceptive pill, which is taken once a week. It is an oral method (I)1 . So, the correct answer is indeed (1) A-III, B-IV, C-II, D-I.
  • 51. Dr. Shivankan Kakkar, MBBS, MD (Gold Medal) 25. Match List I with List II. List I (Type of Joint) List II (Found between) A. Cartilaginous Joint I. Between flat skull bones B. Ball and Socket Joint II. Between adjacent vertebrae in vertebral column C. Fibrous Joint III. Between carpal and metacarpal of thumb D. Saddle Joint IV. Between Humerus and Pectoral girdle Choose the correct answer from the options given below: (1) A-II, B-IV, C-I, D-III (2) A-I, B-IV, C-III, D-II (3) A-II, B-IV, C-III, D-I (4) A-III, B-I, C-II, D-IV Answer (1)
  • 52. Dr. Shivankan Kakkar, MBBS, MD (Gold Medal) • Cartilaginous Joint (A): These are partly movable joints comprising of symphysis or synchondrosis joints. These joints occur only in those regions where the connection between the articulating bones is made up of cartilage1 . They are found between adjacent vertebrae in the vertebral column (II)1 . • Ball and Socket Joint (B): This type of joint allows for the most movement, including rotation. An example of a ball and socket joint is the shoulder joint, which is between the Humerus and the Pectoral girdle (IV)1 . • Fibrous Joint ©: These are joints in which bones are joined by dense connective tissue that is rich in collagen fibers. These joints are also called sutures. The joints between flat skull bones (I) are fibrous joints1 . • Saddle Joint (D): This type of joint allows movement in two directions, like forward and backward or side to side. An example of a saddle joint is the one between the carpal and metacarpal of the thumb (III)1 . So, the correct answer is (1) A-II, B-IV, C-I, D-III.
  • 53. Dr. Shivankan Kakkar, MBBS, MD (Gold Medal) 26. Given below are two statements: Statement I: RNA mutates at a faster rate. Statement II: Viruses having RNA genome and shorter life span mutate and evolve faster. In the light of the above statements, choose the correct answer from the options given below: (1) Both Statement I and Statement II are false. (2) Statement I is true but Statement II is false. (3) Statement I is false but Statement II is true. (4) Both Statement I and Statement II are true. Answer (4)
  • 54. Dr. Shivankan Kakkar, MBBS, MD (Gold Medal) • Statement I: This statement is true. RNA mutates at a faster rate than DNA because RNA is more prone to errors during replication, repair, and recombination. RNA polymerases, the enzymes that copy RNA, lack proofreading mechanisms and have higher error rates than DNA polymerases1 . RNA is also more chemically unstable and reactive than DNA, and can be damaged by various environmental factors1 . • Statement II: This statement is true. Viruses having RNA genome and shorter life span mutate and evolve faster than viruses having DNA genome and longer life span. This is because RNA viruses have higher mutation rates per replication cycle, and shorter life span means more replication cycles per unit of time. Therefore, RNA viruses accumulate more mutations and genetic diversity in a given population, which increases their chances of adaptation and evolution1 . Therefore, the correct answer is (4) Both Statement I and Statement II are true.
  • 55. Dr. Shivankan Kakkar, MBBS, MD (Gold Medal) 27. Given below are two statements: Statement I: In prokaryotes, the positively charged DNA is held with some negatively charged proteins in a region called nucleoid. Statement II: In eukaryotes, the negatively charged DNA is wrapped around the positively charged histone octamer to form nucleosome. In the light of the above statements, choose the correct answer from the options given below: (1) Both Statement I and Statement II are false. (2) Statement I is correct but Statement II is false. (3) Statement I is incorrect but Statement II is true. (4) Both Statement I and Statement II are true. Answer (3)
  • 56. Dr. Shivankan Kakkar, MBBS, MD (Gold Medal) DNA Packaging: Nucleosomes and Chromatin • Statement I: This statement is incorrect. In prokaryotes, the negatively charged DNA is held with some positively charged proteins in a region called the nucleoid. The DNA in prokaryotes is double-stranded and circular, and it is not associated with histones, but with other types of basic proteins that help compact and organize it12345 . • Statement II: This statement is true. In eukaryotes, the negatively charged DNA is wrapped around the positively charged histone octamer to form a nucleosome. A nucleosome is the basic unit of DNA packaging in eukaryotes, and it consists of 146 base pairs of DNA wrapped around a core of eight histone proteins (two copies each of H2A, H2B, H3, and H4)67 . Nucleosomes help condense the DNA into a smaller volume and regulate its accessibility for transcription and replication67 . Therefore, the correct answer is (3) Statement I is incorrect but Statement II is true.
  • 57. Dr. Shivankan Kakkar, MBBS, MD (Gold Medal) 28. Which one of the following symbols represents mating between relatives in human pedigree analysis? Answer (1)
  • 58. Dr. Shivankan Kakkar, MBBS, MD (Gold Medal) Mating between relatives, also known as consanguineous mating, is the mating of individuals who are related by blood or common ancestry1 . Consanguineous mating can increase the risk of genetic disorders, as it reduces the genetic diversity and increases the chances of inheriting recessive alleles12 . In human pedigree analysis, consanguineous mating is represented by a double horizontal line between the symbols of the mating partners3 .
  • 59. Dr. Shivankan Kakkar, MBBS, MD (Gold Medal) 29. Select the correct group/set of Australian Marsupials exhibiting adaptive radiation. (1) Numbat, Spotted cuscus, Flying phalanger (2) Mole, Flying squirrel, Tasmanian tiger cat (3) Lemur, Anteater, Wolf (4) Tasmanian wolf, Bobcat, Marsupial mole Answer (1)
  • 60. Dr. Shivankan Kakkar, MBBS, MD (Gold Medal) Australian marsupials such as Numbat, Spotted cuscus, and Flying phalanger are examples of adaptive radiation, which is the diversification of a single ancestral species into a variety of different forms to exploit different ecological niches. The other options are not correct because they either include placental mammals or non- Australian animals.
  • 61. Dr. Shivankan Kakkar, MBBS, MD (Gold Medal) 30. Which of the following is not a cloning vector? (1) YAC (2) pBR322 (3) Probe (4) BAC Answer (3)
  • 62. Dr. Shivankan Kakkar, MBBS, MD (Gold Medal) A probe is not a cloning vector because it is a short DNA or RNA fragment that is used to detect the presence or location of a specific sequence in a sample. A cloning vector is a small piece of DNA that can be stably maintained in an organism, and into which a foreign DNA fragment can be inserted for cloning purposes1 This link, explains the definition, features and types of cloning vectors, such as plasmids, bacteriophages, phagemids and more.
  • 63. Dr. Shivankan Kakkar, MBBS, MD (Gold Medal) 31. Match List I with List II. List I List II A. Heroin I. Effect on cardiovascular system B. Marijuana II. Slow down body function C. Cocaine III. Painkiller D. Morphine IV. Interfere with transport of dopamine Choose the correct answer from the options given below: (1) A-I, B-II, C-III, D-IV (2) A-IV, B-III, C-II, D-I (3) A-III, B-IV, C-I, D-II (4) A-II, B-I, C-IV, D-III Answer (4)
  • 64. Dr. Shivankan Kakkar, MBBS, MD (Gold Medal) • Heroin (A): Heroin is an opioid that is often used illegally for its euphoric effects. It is known to slow down body functions, which corresponds to II12345 . • Marijuana (B): Marijuana is a psychoactive drug from the Cannabis plant used for medical or recreational purposes. It has various effects on the body, including effects on the cardiovascular system, which corresponds to I678910 . • Cocaine ©: Cocaine is a strong stimulant most frequently used as a recreational drug. It is known for its effect of interfering with the transport of dopamine, a neurotransmitter associated with pleasure and reward, which corresponds to IV111213 . • Morphine (D): Morphine is a pain medication of the opiate variety. It is used to help relieve severe or acute or chronic pain, which corresponds to III2 . The correct matching is A-II, B-I, C-IV, D-III, which is option (4).
  • 65. Dr. Shivankan Kakkar, MBBS, MD (Gold Medal) 32. Which one of the following techniques does not serve the purpose of early diagnosis of a disease for its early treatment? (1) Serum and Urine analysis (2) Polymerase Chain Reaction (PCR) technique (3) Enzyme Linked Immuno-Sorbent Assay (ELISA) technique (4) Recombinant DNA Technology Answer (4)
  • 66. Dr. Shivankan Kakkar, MBBS, MD (Gold Medal) • Serum and Urine analysis: These are conventional diagnostic methods used to detect biomarkers for various diseases. For instance, serum and urine tests can provide early detection of chronic kidney disease1 . • Polymerase Chain Reaction (PCR) technique: PCR is a molecular biology technique that amplifies a small amount of DNA, enabling analysis to be conducted much more easily. It is widely used in medical and biological research labs for the diagnosis of infectious diseases, genetic disorders, and cancers345 . • Enzyme Linked Immuno-Sorbent Assay (ELISA) technique: ELISA is an immunological assay used to measure antibodies, antigens, proteins, and glycoproteins in biological samples. It is commonly used for the diagnosis of various diseases, including HIV infection, pregnancy tests, and measurement of cytokines or soluble receptors in cell supernatant or serum6789 . • Recombinant DNA Technology: This technology involves the introduction of a foreign piece of DNA structure into the genome which contains our gene of interest. It has been used for producing artificial DNA to generate the desired product10 . Among these techniques, Recombinant DNA Technology (option 4) is the one that does not primarily serve the purpose of early diagnosis of a disease for its early treatment.
  • 67. Dr. Shivankan Kakkar, MBBS, MD (Gold Medal) 33. Given below are two statements: one is labelled as Assertion A and the other is labelled as Reason R. Assertion A: Endometrium is necessary for implantation of blastocyst. Reason R: In the absence of fertilization, the corpus luteum degenerates that causes disintegration of endometrium. In the light of the above statements, choose the correct answer from the options given below: (1) Both A and R are true but R is NOT the correct explanation of A. (2) A is true but R is false. (3) A is false but R is true. (4) Both A and R are true and R is the correct explanation of A. Answer (1)
  • 68. Dr. Shivankan Kakkar, MBBS, MD (Gold Medal) • Assertion A: The endometrium, which is the inner lining of the uterus, is indeed necessary for the implantation of a blastocyst1234 . A blastocyst is a ball of cells that forms early in a pregnancy, about five to six days after a sperm fertilizes an egg. It implants in the uterine wall, eventually becoming the embryo and then the fetus1 . If the blastocyst doesn’t implant in the endometrium, pregnancy will not occur15 . • Reason R: In the absence of fertilization, the corpus luteum, a temporary endocrine structure, does degenerate678910 . This leads to a decline in the progesterone level, which causes disintegration of the endometrium leading to menstruation678910 . However, this is not the reason why the endometrium is necessary for the implantation of a blastocyst. So, both A and R are true, but R is NOT the correct explanation of A. Therefore, the correct answer is option (1).
  • 69. Dr. Shivankan Kakkar, MBBS, MD (Gold Medal) 34. Match List I with List II. List I List II A. Gene ‘a’ I. ß-galactosidase B. Gene ‘y’ II. Transacetylase C. Gene ‘i’ III. Permease D. Gene ‘z’ IV. Repressor protein Choose the correct answer from the options given below: (1) A-II, B-III, C-IV, D-I (2) A-III, B-IV, C-I, D-II (3) A-III, B-I, C-IV, D-II (4) A-II, B-I, C-IV, D-III Answer (1)
  • 70. Dr. Shivankan Kakkar, MBBS, MD (Gold Medal) • Gene ‘a’: This gene codes for a transacetylase12 . Transacetylase is an enzyme that transfers an acetyl group from one molecule to another. In the context of the lac operon in E.coli, the ‘a’ gene encodes a transacetylase that transfers an acetyl group from acetyl-CoA to β-galactosides12 . Therefore, Gene ‘a’ corresponds to II. Transacetylase. • Gene ‘y’: This gene codes for permease345 . Permease is a type of protein that aids in the transport of a specific type of molecule across the cell membrane. In the lac operon, the ‘y’ gene codes for β-galactoside permease, a protein that enables the transport of lactose into the cell345 . Therefore, Gene ‘y’ corresponds to III. Permease. • Gene ‘i’: This gene codes for the repressor protein678 . The repressor protein binds to the operator region of the operon and prevents transcription of the downstream genes. In the lac operon, the ‘i’ gene codes for the lac repressor, which binds to the operator region of the lac operon and prevents transcription of the ‘z’, ‘y’, and ‘a’ genes in the absence of lactose678 . Therefore, Gene ‘i’ corresponds to IV. Repressor protein. • Gene ‘z’: This gene codes for β-galactosidase91011 . β-galactosidase is an enzyme that catalyzes the hydrolysis of β-galactosides into monosaccharides. In the lac operon, the ‘z’ gene codes for β-galactosidase, which catalyzes the hydrolysis of lactose into glucose and galactose91011 . Therefore, Gene ‘z’ corresponds to I. ß-galactosidase. The correct matching is A-II, B-III, C-IV, D-I, which corresponds to option (1).
  • 71. Dr. Shivankan Kakkar, MBBS, MD (Gold Medal) 35. Given below are two statements: one is labelled as Assertion A and the other is labelled as Reason R. Assertion A: Nephrons are of two types: Cortical & Juxta medullary, based on their relative position in cortex and medulla. Reason R: Juxta medullary nephrons have short loop of Henle whereas, cortical nephrons have longer loop of Henle. In the light of the above statements, choose the correct answer from the options given below: (1) Both A and R are true but R is NOT the correct explanation of A. (2) A is true but R is false. (3) A is false but R is true. (4) Both A and R are true and R is the correct explanation of A. Answer (2)
  • 72. Dr. Shivankan Kakkar, MBBS, MD (Gold Medal) • Assertion A: Nephrons are indeed of two types: Cortical and Juxta medullary, based on their relative position in the cortex and medulla1234 . Cortical nephrons are located primarily in the renal cortex and make up about 85% of nephrons in the human kidney. Juxtamedullary nephrons have their renal corpuscles close to the medulla, and their loops of Henle extend deep into the renal medulla256 . • Reason R: This statement is incorrect. In fact, it’s the opposite: Juxtamedullary nephrons have a longer loop of Henle which extends deeper into the inner medulla, while cortical nephrons contain a shorter loop of Henle which only extends into the outer region of the renal medulla25789 . A is true but R is false. Therefore, the correct answer is option (2).
  • 73. Dr. Shivankan Kakkar, MBBS, MD (Gold Medal) SECTION B 36. Which of the following is characteristic feature of cockroach regarding sexual dimorphism? (1) Presence of anal styles (2) Presence of sclerites (3) Presence of anal cerci (4) Dark brown body colour and anal cerci Answer (1)
  • 74. Dr. Shivankan Kakkar, MBBS, MD (Gold Medal) 1.Presence of anal styles: Anal styles are present in male cockroaches and absent in females1234 . They are small, thread-like structures that are used during mating3 . The male uses them to grasp the female during copulation and they also play a role in transferring sperm3 . 2.Presence of sclerites: Sclerites are chitinous exoskeleton plates that provide rigidity and protection567 . They are present in all the body segments of both male and female cockroaches56 . 3.Presence of anal cerci: Anal cerci are a pair of filamentous structures that are present in both male and female cockroaches82910 . They are sensory structures that help the cockroach to detect vibrations and other stimuli in their environment910 . 4.Dark brown body colour and anal cerci: The body of the cockroach is covered by a hard chitinous exoskeleton that is brown in colour11 . Both male and female cockroaches have anal cerci82910 . The correct answer is option (1) - Presence of anal styles, as this is the only feature that is sexually dimorphic, i.e., different between male and female cockroaches1234 .
  • 75. Dr. Shivankan Kakkar, MBBS, MD (Gold Medal) 37. Select the correct statements. A. Tetrad formation is seen during Leptotene. B. During Anaphase, the centromeres split and chromatids separate. C. Terminalization takes place during Pachytene. D. Nucleolus, Golgi complex and ER are reformed during Telophase. E. Crossing over takes place between sister chromatids of homologous chromosome. Choose the correct answer from the options given below: (1) B and D only (2) A,C and E only (3) B and E only (4) A and C only Answer (1)
  • 76. Dr. Shivankan Kakkar, MBBS, MD (Gold Medal) • Statement A: Tetrad formation is seen during Leptotene. This statement is incorrect. Tetrad formation, also known as synapsis, occurs during the Zygotene stage of Prophase I of meiosis1234 . • Statement B: During Anaphase, the centromeres split and chromatids separate. This statement is correct. During Anaphase, the centromeres of each chromosome split, and the sister chromatids separate and move to opposite poles56789 . • Statement C: Terminalization takes place during Pachytene. This statement is incorrect. Terminalization, which refers to the movement of chiasmata (sites of crossing over) towards the ends (terminals) of the homologous chromosomes, occurs during Diplotene, not Pachytene10113 . • Statement D: Nucleolus, Golgi complex and ER are reformed during Telophase. This statement is correct. During Telophase, the nuclear envelope reforms around each set of separated chromosomes. The nucleolus, Golgi complex, and endoplasmic reticulum (ER) are also reformed within the newly formed nuclei121341415 . • Statement E: Crossing over takes place between sister chromatids of homologous chromosome. This statement is incorrect. Crossing over is the exchange of genetic material between non-sister chromatids of homologous chromosomes during meiosis, which results in new allelic combinations in the daughter cells16171819 . The correct statements are B and D only, which corresponds to option (1).
  • 77. Dr. Shivankan Kakkar, MBBS, MD (Gold Medal) 38. Which of the following statements are correct? A. An excessive loss of body fluid from the body switches off osmoreceptors. B. ADH facilitates water reabsorption to prevent diuresis. C. ANF causes vasodilation. D. ADH causes increase in blood pressure. E. ADH is responsible for decrease in GFR. Choose the correct answer from the options given below: (1) B, Cand D only (2) A, B and E only (3) C, D and E only (4) A and B only Answer (1)
  • 78. Dr. Shivankan Kakkar, MBBS, MD (Gold Medal) • Statement A: An excessive loss of body fluid from the body switches off osmoreceptors. This statement is incorrect. Osmoreceptors are specialized cells that detect changes in osmotic pressure and regulate the release of antidiuretic hormone (ADH). Excessive loss of body fluid would lead to increased osmotic pressure and activate osmoreceptors, not switch them off1234 . • Statement B: ADH facilitates water reabsorption to prevent diuresis. This statement is correct. Antidiuretic hormone (ADH), also known as vasopressin, acts on the kidneys to increase water reabsorption. It helps prevent excessive water loss and promotes water retention in the body56718 . • Statement C: ANF causes vasodilation. This statement is correct. Atrial natriuretic peptide (ANF) is a hormone released by the heart in response to increased blood volume and pressure. It promotes vasodilation, which leads to the relaxation of blood vessels and increased excretion of sodium and water91011 . • Statement D: ADH causes increase in blood pressure. This statement is correct. ADH acts on the blood vessels, causing vasoconstriction. This constriction increases peripheral vascular resistance, which can contribute to an increase in blood pressure6121314 . • Statement E: ADH is responsible for decrease in GFR. This statement is incorrect. ADH does not directly influence the glomerular filtration rate (GFR). It primarily affects water reabsorption in the renal tubules11516 . The correct statements are B, C, and D only, which corresponds to option (1).
  • 79. Dr. Shivankan Kakkar, MBBS, MD (Gold Medal) 39. Given below are two statements: Statement I : During G0 phase of cell cycle, the cell is metabolically inactive. Statement II : The centrosome undergoes duplication during S phase of interphase. In the light of the above statements, choose the most appropriate answer from the options given below: (1) Both Statement I and Statement II are incorrect. (2) Statement I is correct but Statement II is incorrect. (3) Statement I is incorrect but Statement II is correct. (4) Both Statement I and Statement II are correct Answer (3)
  • 80. Dr. Shivankan Kakkar, MBBS, MD (Gold Medal) • Statement I: During G0 phase of cell cycle, the cell is metabolically inactive. This statement is incorrect. While the G0 phase is considered a quiescent phase where cells are not actively dividing, many cells in G0 are still metabolically active and can carry out essential functions12 . • Statement II: The centrosome undergoes duplication during S phase of interphase. This statement is correct. The centrosome starts duplicating just before, or at, the onset of S phase and the two newly formed centrosomes participate in the assembly and organization of the mitotic spindle345 . So, Statement I is incorrect but Statement II is correct. Therefore, the correct answer is option (3).
  • 81. Dr. Shivankan Kakkar, MBBS, MD (Gold Medal) 40. Which one of the following is NOT an advantage of inbreeding? (1) It exposes harmful recessive genes but are eliminated by selection. (2) Elimination of less desirable genes and accumulation of superior genes takes place due to it. (3) It decreases the productivity of inbred population, after continuous inbreeding. (4) It decreases homozygosity. Answer (3)
  • 82. Dr. Shivankan Kakkar, MBBS, MD (Gold Medal) • It exposes harmful recessive genes but are eliminated by selection. Inbreeding can lead to the exposure of harmful recessive genes. These harmful genes can be eliminated over time through the process of natural selection12 . • Elimination of less desirable genes and accumulation of superior genes takes place due to it. Inbreeding can lead to the elimination of less desirable genes and the accumulation of superior genes. This happens as individuals with similar genetic backgrounds mate, reducing the frequency of less desirable genes and concentrating desirable genes3456 . • It decreases the productivity of inbred population, after continuous inbreeding. Continuous inbreeding can lead to a decrease in the productivity of the inbred population. This is due to the increased expression of harmful recessive genes, a phenomenon known as inbreeding depression7891011 . . Continuous inbreeding can decrease the productivity of the inbred population, which is a disadvantage, not an advantage. • It decreases homozygosity. This statement is incorrect. Inbreeding actually increases homozygosity, not decreases it. Inbreeding leads to an increase in homozygosity because it involves mating between closely related individuals781213 . The answer is (3) because it is not an advantage of inbreeding.
  • 83. Dr. Shivankan Kakkar, MBBS, MD (Gold Medal) 41. Match List I with List II. List I List II A. Logistic growth I. Unlimited resource availability condition B. Exponential growth II. Limited resource availability condition C. Expanding age pyramid III. The percent individuals of pre-reproductive age is largest followed by reproductive and post reproductive age groups D. Stable age pyramid IV.The percent individuals of pre-reproductives and reproductive age group are same Choose the correct answer from the options given below: (1) A-II, B-III, C-I, D-IV (2) A-II, B-IV, C-I, D-III (3) A-II, B-IV, C-III, D-I (4) A-II, B-I, C-III, D-IV Answer (4)
  • 84. Dr. Shivankan Kakkar, MBBS, MD (Gold Medal) • A. Logistic growth matches with II. Limited resource availability condition. Logistic growth occurs when resources are limited, causing the population growth rate to decrease as the population size approaches the environment’s carrying capacity1234 . • B. Exponential growth matches with I. Unlimited resource availability condition. Exponential growth occurs when resources are abundant, allowing the population to grow rapidly567 . • C. Expanding age pyramid matches with III. The percent individuals of pre- reproductive age is largest followed by reproductive and post reproductive age groups. An expanding age pyramid is characterized by a larger percentage of individuals in younger age groups, indicating high fertility rates and lower life expectancies89101112 . • D. Stable age pyramid matches with IV. The percent individuals of pre- reproductives and reproductive age group are same. A stable age pyramid shows a somewhat equal proportion of the population in each age group, indicating a stable population13141516 . The answer is (4) because it correctly matches each item in List I with the corresponding item in List II.
  • 85. Dr. Shivankan Kakkar, MBBS, MD (Gold Medal) 42. Which of the following are NOT under the control of thyroid hormone? A. Maintenance of water and electrolyte balance B. Regulation of basal metabolic rate C. Normal rhythm of sleep-wake cycle D. Development of immune system E. Support the process of RBCs formation Choose the correct answer from the options given below: (1) B and C only (2) C and D only (3) D and E only (4) A and D only Answer (2)
  • 86. Dr. Shivankan Kakkar, MBBS, MD (Gold Medal) A. Maintenance of water and electrolyte balance: This is controlled by the thyroid hormone12 . B. Regulation of basal metabolic rate: The thyroid hormone plays a crucial role in regulating the body’s metabolism12 . C. Normal rhythm of sleep-wake cycle: While the thyroid hormone can influence energy levels, it does not directly control the sleep-wake cycle12 . D. Development of immune system: The thyroid hormone does not directly control the development of the immune system12 . E. Support the process of RBCs formation: The thyroid hormone does play a role in the differentiation and development of cells2 , which could influence RBC formation. So, the options C and D (the sleep-wake cycle and the development of the immune system) are not directly under the control of the thyroid hormone, which corresponds to answer (2). Therefore, the given answer is correct.
  • 87. Dr. Shivankan Kakkar, MBBS, MD (Gold Medal) 43. The unique mammalian characteristics are: (1) hairs, pinna and mammary glands (2) hairs, pinna and indirect development (3) pinna, monocondylic skull and mammary glands (4) hairs, tympanic membrane and mammary glands Answer (1)
  • 88. Dr. Shivankan Kakkar, MBBS, MD (Gold Medal) 1.Hairs, pinna, and mammary glands: All mammals have hair at some stage of their life cycle1 . The pinna is the visible part of the ear that resides outside of the head, and it’s a feature that most mammals have. Mammary glands, which produce milk to feed young, are a defining characteristic of mammals1 . 2.Hairs, pinna, and indirect development: While hairs and pinna are mammalian characteristics, indirect development is not a unique mammalian characteristic. Many animals, not just mammals, undergo indirect development. 3.Pinna, monocondylic skull, and mammary glands: While pinna and mammary glands are mammalian characteristics, a monocondylic skull (a skull with one occipital condyle) is not a unique characteristic of mammals. In fact, mammals typically have a dicondylic skull (a skull with two occipital condyles)1 . 4.Hairs, tympanic membrane, and mammary glands: While hairs and mammary glands are mammalian characteristics, the presence of a tympanic membrane (eardrum) is not unique to mammals. Many other animals, including birds and reptiles, also have tympanic membranes. So, the only option that lists exclusively mammalian characteristics is option (1): hairs, pinna, and mammary glands. Therefore, the given answer is correct.
  • 89. Dr. Shivankan Kakkar, MBBS, MD (Gold Medal) 44. Which of the following statements are correct regarding skeletal muscle? A. Muscle bundles are held together by collagenous connective tissue layer called fascicle. B. Sarcoplasmic reticulum of muscle fibre is a store house of calcium ions. C. Striated appearance of skeletal muscle fibre is due to distribution pattern of actin and myosin proteins. D. M line is considered as functional unit of contraction called sarcomere. Choose the most appropriate answer from the options given below: (1) B and C only (2) A, C and D only (3) C and D only (4) A, B and Conly Answer (1)
  • 90. Dr. Shivankan Kakkar, MBBS, MD (Gold Medal) • Statement A is incorrect because the fascicle is not the connective tissue layer, but the bundle of muscle fibres. The connective tissue layer that surrounds the fascicle is called the perimysium. • Statement B is correct because the sarcoplasmic reticulum is a specialized endoplasmic reticulum that stores and releases calcium ions during muscle contraction and relaxation. • Statement C is correct because the striated appearance of skeletal muscle fibre is due to the alternating bands of thin filaments (made of actin) and thick filaments (made of myosin) that form the myofibrils. • Statement D is correct because the M line is the middle point of a sarcomere, which is the functional unit of contraction in skeletal muscle. The sarcomere is the region between two Z lines, which mark the boundaries of a myofibril segment.
  • 91. Dr. Shivankan Kakkar, MBBS, MD (Gold Medal) 45. Which of the following statements are correct? A. Basophils are most abundant cells of the total WBCs B. Basophils secrete histamine, serotonin and heparin C. Basophils are involved in inflammatory response D. Basophils have kidney shaped nucleus E. Basophils are agranulocytes Choose the correct answer from the options given below: (1) C and E only (2) B and C only (3) A and B only (4) D and E only Answer (2)
  • 92. Dr. Shivankan Kakkar, MBBS, MD (Gold Medal) The correct answer is (2) B and C only. Here is an explanation: • Statement A is incorrect because basophils are the least abundant cells of the total WBCs, accounting for only about 0.5% of the circulating leukocytes12 . • Statement B is correct because basophils secrete histamine, which is a vasodilator that increases blood flow to the tissues, and heparin, which is an anticoagulant that prevents blood clotting12 . Basophils also secrete serotonin, which is a neurotransmitter that modulates pain and mood3 . • Statement C is correct because basophils are involved in inflammatory response, especially in allergic reactions and parasitic infections12 . Basophils release histamine and other mediators that cause symptoms such as itching, swelling, and sneezing12 . • Statement D is incorrect because basophils have a lobed nucleus, not a kidney- shaped nucleus12 . The nucleus of basophils is often obscured by the large cytoplasmic granules that stain dark purple or black12 . • Statement E is incorrect because basophils are granulocytes, not agranulocytes12 . Granulocytes are a type of white blood cells that have granules in their cytoplasm, whereas agranulocytes are a type of white blood cells that lack granules in their cytoplasm12 .
  • 93. Dr. Shivankan Kakkar, MBBS, MD (Gold Medal) 46. Which one of the following is the sequence on corresponding coding strand, if the sequence on mRNA formed is as follows 5’AUCGAUCGAUCGAUCGAUCGAUCG AUCG 3’? (1) 3’ UAGCUAGCUAGCUAGCUAGCUAGCUAGC 5’ (2) 5’ ATCGATCGATCGATCGATCGATCGATCG 3’ (3) 3’ ATCGATCGATCGATCGATCGATCGATCG 5’ (4) 5’ UAGCUAGCUAGCUAGCUAGCUAGCUAGC 3’ Answer (2)
  • 94. Dr. Shivankan Kakkar, MBBS, MD (Gold Medal) The correct answer is (2) 5’ ATCGATCGATCGATCGATCGATCGATCG 3’. Here is an explanation: • The coding strand is the DNA strand whose base sequence is identical to the base sequence of the mRNA transcript produced, except for the replacement of thymine (T) in DNA with uracil (U) in mRNA12 . • The coding strand is also presented in the 5’ to 3’ direction, which is the same direction as the mRNA transcript12 . • Therefore, to find the sequence of the coding strand, we just need to replace U with T in the mRNA sequence, and keep the same direction. • The mRNA sequence is 5’AUCGAUCGAUCGAUCGAUCGAUCG AUCG 3’, so the coding strand sequence is 5’ATCGATCGATCGATCGATCGATCG ATCG 3’.
  • 95. Dr. Shivankan Kakkar, MBBS, MD (Gold Medal) 47. The parts of human brain that helps in regulation of sexual behaviour, expression of excitement, pleasure, rage, fear etc. are: (1) Corpora quadrigemina and hippocampus (2) Brain stem and epithalamus (3) Corpus callosum and thalamus (4) Limbic system and hypothalamus Answer (4)
  • 96. Dr. Shivankan Kakkar, MBBS, MD (Gold Medal) The correct answer is (4) Limbic system and hypothalamus. Here is an explanation: • The limbic system is a complex network of brain structures that includes the amygdala, hippocampus, cingulate gyrus, and other parts12 . The limbic system is involved in regulating emotions, motivation, memory, and learning12 . It also plays a key role in sexual behaviour, as it mediates the reward and pleasure associated with mating34 . • The hypothalamus is a small but important region of the brain that lies below the thalamus and above the pituitary gland12 . The hypothalamus is responsible for maintaining the body’s homeostasis, or internal balance, by regulating various physiological functions such as temperature, hunger, thirst, sleep, and circadian rhythms12 . The hypothalamus also controls the secretion of hormones from the pituitary gland, which in turn affect the reproductive system, growth, metabolism, and stress response12 . The hypothalamus also influences sexual behaviour, as it integrates sensory and hormonal signals and coordinates the sexual arousal and orgasm34 .
  • 97. Dr. Shivankan Kakkar, MBBS, MD (Gold Medal) 48. Match List I with List II. List I List II A. Mast cells I. Ciliated epithelium B. Inner surface of bronchiole II. Areolar connective tissue C. Blood III. Cuboidal epithelium D. Tubular parts of nephron IV. Specialised connective tissue Choose the correct answer from the options give below: (1) A-II, B-III, C-I, D-IV (2) A-II, B-I, C-IV, D-III (3) A-III, B-IV, C-II, D-I (4) A-I, B-II, C-IV, D-III Answer (2)
  • 98. Dr. Shivankan Kakkar, MBBS, MD (Gold Medal) • A. Mast cells are a type of immune cell that are found in connective tissue, especially in the skin, respiratory tract, and gastrointestinal tract. They release histamine and other mediators that cause allergic reactions and inflammation. Mast cells are found in areolar connective tissue, which is a loose type of connective tissue that fills the spaces between organs and tissues12 . • B. The inner surface of bronchiole is lined by ciliated epithelium, which is a type of epithelial tissue that has hair-like projections called cilia on its surface. The cilia help to move mucus and trapped particles out of the respiratory tract34 . • C. Blood is a specialized connective tissue that consists of plasma, blood cells, and platelets. It circulates throughout the body and transports oxygen, nutrients, hormones, and waste products. Blood is classified as a connective tissue because it originates from the mesoderm and has a matrix (plasma) that supports the cells56 . • D. The tubular parts of nephron are the structures in the kidney that filter the blood and produce urine. They include the proximal tubule, the loop of Henle, the distal tubule, and the collecting duct. The tubular parts of nephron are lined by cuboidal epithelium, which is a type of epithelial tissue that has cube-shaped cells. The cuboidal epithelium facilitates the reabsorption and secretion of water and solutes in the nephron78 .
  • 99. Dr. Shivankan Kakkar, MBBS, MD (Gold Medal) 49. In cockroach, excretion is brought about by- A. Phallic gland B. Uricose gland C. Nephrocytes D. Fat body E. Collaterial glands (1) A, B and E only (2) B, Cand D only (3) B and D only (4) A and E only Answer (2)
  • 100. Dr. Shivankan Kakkar, MBBS, MD (Gold Medal) • A. Phallic gland: This is a gland that is associated with the male reproductive system of cockroaches. It secretes a substance that forms the spermatophore, which is a capsule that contains the sperm. The phallic gland is not involved in excretion1 . • B. Uricose gland: This is a gland that is located in the hindgut of cockroaches. It secretes uric acid, which is the main nitrogenous waste product of cockroaches. The uricose gland helps to conserve water and maintain osmotic balance. The uricose gland is involved in excretion23 . • C. Nephrocytes: These are cells that are found in the haemocoel of cockroaches. They are phagocytic cells that engulf and store waste materials, such as uric acid crystals, from the haemolymph. They also help to regulate the pH and ion concentration of the haemolymph. Nephrocytes are involved in excretion24 . • D. Fat body: This is a tissue that is distributed throughout the body cavity of cockroaches. It consists of various types of cells, such as trophocytes, mycetocytes, and urocytes. The fat body performs several functions, such as storage of nutrients, synthesis of proteins and lipids, and detoxification of toxins. The fat body also contains urocytes, which are cells that store uric acid in vacuoles. The fat body is involved in excretion2 . • E. Collaterial glands: These are glands that are associated with the female reproductive system of cockroaches. They secrete a substance that forms the ootheca, which is a protective case that contains the eggs. The collaterial glands are not involved in excretion1 .
  • 101. Dr. Shivankan Kakkar, MBBS, MD (Gold Medal) 50. Select the correct statements with reference to chordates. A. Presence of a mid-dorsal, solid and double nerve cord. B. Presence of closed circulatory system. C. Presence of paired pharyngeal gill slits. D. Presence of dorsal heart E. Triploblastic pseudocoelomate animals. Choose the correct answer from the options given below: (1) B and C only (2) B,D and E only (3) C, D and E only (4) A, Cand D only Answer (1)
  • 102. Dr. Shivankan Kakkar, MBBS, MD (Gold Medal) • Statement A is incorrect because chordates have a mid-dorsal, hollow and single nerve cord, not a solid and double nerve cord12 . The nerve cord is located above the notochord and develops into the brain and spinal cord in vertebrates12 . • Statement B is correct because chordates have a closed circulatory system, which means that the blood is confined within blood vessels and pumped by a heart12 . The circulatory system transports oxygen, nutrients, hormones, and waste products throughout the body12 . • Statement C is correct because chordates have paired pharyngeal gill slits, which are openings in the pharynx (throat) that connect to the external environment12 . The gill slits function as a filter-feeding device in some invertebrate chordates, and as a respiratory organ in aquatic vertebrates12 . • Statement D is incorrect because chordates have a ventral heart, not a dorsal heart12 . The heart is located below the pharynx and pumps blood through the circulatory system12 . • Statement E is incorrect because chordates are triploblastic coelomate animals, not pseudocoelomate animals12 . This means that they have three germ layers (ectoderm, mesoderm, and endoderm) and a true body cavity (coelom) that is lined by mesoderm12 .